Land Titles and Deeds - Study Guide (Draft)

May 28, 2018 | Author: Clay Delgado | Category: Patent Application, Property, Classified Information, Adverse Possession, Ownership
Share Embed Donate


Short Description

Download Land Titles and Deeds - Study Guide (Draft)...

Description

LAND TITLES AND DEEDS Study Guide Clayton M. Delgado I.

REGALIAN DOCTRINE and The CONSTITUTION

A. Regalian doctrine, which is embodied in our Constitution, all lands of the public domain belong to the State, which is the source of any asserted right to any ownership of land. All lands not appearing to be clearly within private ownership are presumed to belong to the State. Accordingly, public lands not shown to have been reclassified or released as alienable agricultural land or alienated to a private person by the State remain part of the inalienable public domain. Unless public land is shown to have been reclassified as alienable or disposable to a private person by the State, it remains part of the inalienable public domain. Property of the public domain is beyond the commerce of man and not susceptible of private appropriation and acquisitive prescription. Occupation thereof in the concept of owner no matter how long cannot ripen into ownership and be registered as a title. The burden of proof in overcoming the presumption of State ownership of the lands of the public domain is on the person applying for registration (or claiming ownership), who must prove that the land subject of the application is alienable or disposable. To overcome this presumption, incontrovertible evidence must be established that the land subject of the application (or claim) is alienable or disposable.

B. Art. XII, Sec. 2, 1987 Constitution “All lands of the public domain, waters, minerals, coal, petroleum, and other mineral oils, all

forces of potential energy, fisheries, forests or timber, wildlife, flora and fauna, and other natural resources are owned by the State. With the exception of agricultural lands, all other natural resources shall not be alienated. The exploration, development, and utilization of natural resources shall be under the full control and supervision of the State. The State may directly undertake such activities, or it may enter into co-production, joint venture, or production-sharing agreements with Filipino citizens, or corporations or associations at least sixty per centum of whose capital is owned by such citizens. Such agreements may be for a period not exceeding twenty-five years, renewable for not more than twenty-five years, and under such terms and conditions as may be provided by law. In cases of water rights for irrigation, water supply fisheries, or industrial uses other than the development of water power, beneficial beneficial use may be the measure and limit of the grant.”

C. Case Law 1.  Pedro Lee Hong Hong Hok Vs. Aniano Aniano David  DOCTRINE: Imperium distinguished from dominum. The government authority possessed by the state which is appropriately embraced in the concept of sovereignty comes under the heading of imperium; and its capacity to own or acquire property under dominium. The use of this term is appropriate with reference to lands held by the state in its proprietary character. In such capacity, it may provide for the exploitation and use of lands and other natural resources, including their disposition, except as limited by the Constitution. FACTS -

Petitioners in this appeal by certiorari seek to reverse a decision of respondent Court of Appeals affirming a lower court judgment dismissing their complaint to have the Torrens Title of respondent Aniano David declared null and void. According to the Stipulation of Facts, since the filing of the sales application of Aniano David and during all the proceedings in connection with said application, up to the actual issuance of the sales patent in his favor, the plaintiffs appellants did not put up any opposition or adverse claim thereto. David acquired lawful title pursuant to his miscellaneous sales application in accordance with which an order of award and for issuance of a sales patent was made by the Director of Lands on June 18, 1958, covering Lot 2892 containing an area of 226 square meters, which is a portion of Lot 2863 of the Naga Cadastre, On the basis of the order of award of the Director of Lands the Undersecretary of Agriculture and Natural Resources issued on August 26, 1959, Miscellaneous Sales Patent No. V-1209 pursuant to which OCT No. 510 was issued by the Register of Deeds of Naga City to defendant-appellee Aniano David on October 21, 1959.

ISSUE -

Can petitioners bring an action to cancel a void certificate of title issued pursuant to a void patent?

RULING -

NO. Only the Government, represented by the Director of Lands, or the Secretary of Agriculture and Natural Resources, can bring an action to cancel a void certificate of title issued pursuant to a void patent Whether the grant was in conformity with the law or not is a question which the government may raise, but until it is raised by the government and Page | 1

set aside, the defendant cannot question it. The legality of the grant is a question between the grantee and the government.  2.  Republic of the the Philippines Philippines Vs. Alexandra Alexandra Lao FACTS Lao filed before the RTC of Tagaytay City application for registration of a parcel of land. She allegedly acquired the land by purchase from the siblings Raymundo Noguera and Ma. Victoria Valenzuela who inherited it from Generosa Medina. The latter, in turn, inherited the land from her father, Jose Medina, who acquired the same from Edilberto Perido by transfer. She prayed that the land be registered in her name under Commonwealth Act 141 (Public Land Act) based on her and her predecessor-in-interests’ open, public, actual, continuous, exclusive, notorious and adverse possession and occupancy under bona fide claim of ownership for more than thirty (30) years. She presented witnesses and evidence constituting of deed of sale, survey plan, the technical description of property and tax declarations in her and her predecessors’ names. The court approved the application. The

petitioner represented by the Solicitor General appealed the decision before the CA which reaffirmed the lower court decision, hence this petition for review before the SC. The petitioner contends that there is no sufficient evidence to warrant the issuance of the title to the respondent as she fails to comply with the required periods and acts of possession mandated by law and her failure to prove that the land is alienable and disposable land of the public domain. ISSUE Whether or not the respondent sufficiently provided evidence that she meets the qualifications required by law on the manner of possession (continuous, adverse, notorious, etc..) and the period of time (30 years) necessary to have a bonafide claim of ownership under C.A. 141? Whether or not respondent was able to show that the land subject of her application was disposable and alienable land of the public domain? RULING -

The court held that Commonwealth Act 141 requires that before one can register his title over a parcel of land, the applicant must show that he, by himself or through his predecessors-in-interest, has been in open, continuous, exclusive and notorious possession and occupation of the subject land under a bona fide claim of ownership since June 12, 1945 or earlier; in adverse possession over the land for at least 30 years and the land subject of the application is alienable and disposable land of the public domain. Petitioner was right to contend that the respondent did not prove by incontrovertible evidence that she possessed the property in the manner and time required by law. She did not provide the exact period when her predecessors-in-interest started occupying the property. No extrajudicial settlement of the property from its previous owners was shown and she did not show any relationship between the parties where she obtained her deed of sale. She further did not present any certification from appropriate government agency to show that the property is re-classified as disposable and alienable land of the public domain. It is incumbent for an applicant of a land registration to provide these incontrovertible evidences to support her claim for her application. In the absence of these evidences, her application shall fail. Hence the petition was granted and her application was denied. 3.  Republic of the the Philippines Philippines Vs. Ludolfo Ludolfo Munuz FACTS -

On June 14, 1996, Munoz filed an application for registration of title of a parcel of residential land before the RTC of Albay. Munoz alledgedly acquired the property through donation from her parents, and his parents and predecessors in interest have been in possession of the property since time immemorial for more than 70 years. The residential lot originally owned and possessed by Paulino Pulvinar and Geronimo Lozada, who subsequently both sold their share of unregistered land to the parents of Munoz. The Republic of the Philippines through the OSG opposed the application, among his contentions were that muniments of title, tax payments and receipts of applications do not Page | 2

set aside, the defendant cannot question it. The legality of the grant is a question between the grantee and the government.  2.  Republic of the the Philippines Philippines Vs. Alexandra Alexandra Lao FACTS Lao filed before the RTC of Tagaytay City application for registration of a parcel of land. She allegedly acquired the land by purchase from the siblings Raymundo Noguera and Ma. Victoria Valenzuela who inherited it from Generosa Medina. The latter, in turn, inherited the land from her father, Jose Medina, who acquired the same from Edilberto Perido by transfer. She prayed that the land be registered in her name under Commonwealth Act 141 (Public Land Act) based on her and her predecessor-in-interests’ open, public, actual, continuous, exclusive, notorious and adverse possession and occupancy under bona fide claim of ownership for more than thirty (30) years. She presented witnesses and evidence constituting of deed of sale, survey plan, the technical description of property and tax declarations in her and her predecessors’ names. The court approved the application. The

petitioner represented by the Solicitor General appealed the decision before the CA which reaffirmed the lower court decision, hence this petition for review before the SC. The petitioner contends that there is no sufficient evidence to warrant the issuance of the title to the respondent as she fails to comply with the required periods and acts of possession mandated by law and her failure to prove that the land is alienable and disposable land of the public domain. ISSUE Whether or not the respondent sufficiently provided evidence that she meets the qualifications required by law on the manner of possession (continuous, adverse, notorious, etc..) and the period of time (30 years) necessary to have a bonafide claim of ownership under C.A. 141? Whether or not respondent was able to show that the land subject of her application was disposable and alienable land of the public domain? RULING -

The court held that Commonwealth Act 141 requires that before one can register his title over a parcel of land, the applicant must show that he, by himself or through his predecessors-in-interest, has been in open, continuous, exclusive and notorious possession and occupation of the subject land under a bona fide claim of ownership since June 12, 1945 or earlier; in adverse possession over the land for at least 30 years and the land subject of the application is alienable and disposable land of the public domain. Petitioner was right to contend that the respondent did not prove by incontrovertible evidence that she possessed the property in the manner and time required by law. She did not provide the exact period when her predecessors-in-interest started occupying the property. No extrajudicial settlement of the property from its previous owners was shown and she did not show any relationship between the parties where she obtained her deed of sale. She further did not present any certification from appropriate government agency to show that the property is re-classified as disposable and alienable land of the public domain. It is incumbent for an applicant of a land registration to provide these incontrovertible evidences to support her claim for her application. In the absence of these evidences, her application shall fail. Hence the petition was granted and her application was denied. 3.  Republic of the the Philippines Philippines Vs. Ludolfo Ludolfo Munuz FACTS -

On June 14, 1996, Munoz filed an application for registration of title of a parcel of residential land before the RTC of Albay. Munoz alledgedly acquired the property through donation from her parents, and his parents and predecessors in interest have been in possession of the property since time immemorial for more than 70 years. The residential lot originally owned and possessed by Paulino Pulvinar and Geronimo Lozada, who subsequently both sold their share of unregistered land to the parents of Munoz. The Republic of the Philippines through the OSG opposed the application, among his contentions were that muniments of title, tax payments and receipts of applications do not Page | 2

constitute competent and sufficient evidence of bonafide acquisition.and that the parcel applied for is part of the public domain and not subject to private appropriation. The Regional Trial Court rendered decision in favour of Munoz, noting that the a report submitted by the Director of Lands, that as per records of the Land Management Bureau in Manila the subject property is covered by Free Patent Application No. 10-2-664 of Anastacia Vitero, mother of Munoz. The Court of Appeals affirmed the decision of the RTC. ISSUE -

Whether or not the failure to present the original tracing cloth plan is fatal

omission which necessarily affected the trial court’s jurisdiction.

Whether or not in proving the alienable and disposable nature of the property there has to be a certification from the DENR and Community Environment and Natural Resources Office. RULING As to the first issue, it bears stressing that the constructive seizure of land accomplished by posting of notices and processes upon all persons mentioned in notices by means of publication and sending copies to said persons by registered mail in effect gives the court jurisdiction over the lands sought to be registered. It is true that the best evidence to identify a piece of land for registration purposes is the original tracing cloth plan from the Bureau of Lands, in this case however, the presentation of the original tracing cloth plan may be dispensed with since the blue print copy of the survey plan approved by the Bureau of Lands and the technical descriptions duly verified and approved by the Director of lands were presented and provide sufficient identification. As to the second issue, the CA ruled that Munoz need not adduce documentary proofs for the property to be declared alienable and disposable because of the fact that it had once been covered by Free Patent application in the name of the mother of Munoz, which was unfortunately not acted upon by the proper authorities. This court however cannot sustain this argument. As well settled in jurisprudence, jurisprudence, it is indispensable that the person claiming title to public land should show a positive act of the government such as presidential proclamation, executive order, administrative action, investigation reports of Bureau of Lands investigations, legislative act or a statue or certification from the government that the land applied for is alienable and disposable which the respondent failed to secure. The court cannot approve the application for registration due to failure to prove that the land is alienable and disposable. II.

Limitations, and Exclusion from, the Regalian Doctrine

A. Constitutional Limitation By virtue of the State’s power of ownership over the public domain, it could thereby exercise its

rights of ownership, which include the power to exploit, develop and alienate such natural resources. The Constitution, Constitution, however, provides for specific limitations to such power. These are enumerated in Sections 2 and 3 of Article XII. Because we are now only concerned with the Constitutional limitations on the power of the State to dispose of lands of the public domain, we note the following specific provisions: “SECTION 2. All lands of the public domain . . . are owned owned by the State. With the exception of agricultural lands, all other natural resources shall not be alienated. . .” “SECTION 3. Lands of the public public domain are classified into agricultural, forest forest or timber,

mineral lands, and national parks. Agricultural lands of the public domain may be further classified by law according to the uses which they may be devoted. Alienable lands of the public domain shall be limited to agricultural lands. Private corporations or associations may not hold such alienable lands of the public domain except by lease, for a period not exceeding twenty-five years, renewable for not more than twenty-five years, and not to exceed one thousand hectares in area. Citizens of the Philippines may lease not more than five hundred hectares, or acquire not more than twelve hectares thereof by purchase, homestead, or grant. Taking into account the requirements of conservation, ecology, and development, and subject to the requirements of agrarian reform, the Congress shall determine, by law, the size of lands Page | 3

of the public domain which may be acquired, developed, held, or leased and the conditions therefor.”

Thus, with reference to the Regalian Doctrine, these provisions provide that while no public land can be acquired by private persons without any grant, express or implied, from the government, only alienable and disposable lands of the public domain may nonetheless be subject of such grant. Furthermore, the grantees of such land, even if alienable, must similarly comply with the citizenship requirements prescribed by the Constitution. It must also be noted that notwithstanding the application of the Regalian Doctrine, the colonizers applying the doctrine did not intend to strip the natives of their ownership of lands already belonging to them. This was the ruling in the landmark case of Cariño v. Insular  Government, 41 Phil. 935 (1909), where the Supreme Court said: “when, as far back as testimony

or memory goes, the land has been held by individuals under a claim of private ownership, it will be presumed to have been held in the same way from before the Spanish conquest and never have been public land.” Consequently, such land, if not owned by the State at the time of

the Spanish conquest, could not have been ceded by Spain to the United States through the Treaty of Paris, and later, to the Philippine Government by the time of the Commonwealth. The doctrine of immemorial possession set forth in Cariño was also recognized in the opinion defending the constitutionality Republic Act No. 8371 or Indigenous People’s Rights Act (IPRA). In the case of Cruz v. Executive Secretary, G.R. No. 135385, 6 December 2000, Republic

Act No. 8371 said was assailed as unconstitutional on the ground that it deprived the state of ownership over lands of public domain and natural resources contained therein. The votes were deadlocked at 7-7 which meant that the validity of the IPRA was upheld. The opinion defending the validity of the IPRA held that ancestral domain and ancestral lands are private and belong to the indigenous people. B. ANCESTRAL LANDS Cariño v. Insular Government 41 Phil. 936

FACTS: An Igorot applied to the Philippine court for registration of a certain parcel of land. The plaintiff and his ancestors held the land since time immemorial. The Philippine government opposed such application saying that there is no prescription against the crown, and even if there was, the land is not registered therefore it is public land by virtue of the Decree of June 25, 1880 which required registration for good title; and because of such the U.S. is the owner of the property by succeeding Spain by virtue of the Treaty of Paris. ISSUE: WON the land is public or not. HELD: It is not public. In this case, every presumption must be in favor of the individual and against the government. Therefore, it can be “proper and sufficient to say that when, as far back

as testimony or memory goes, the land has been held by individuals under a claim of private ownership, it will be presumed to have been held in the same way from before the Spanish conquest, and never to have been public land.” With this, it can be inferred that immemorial possession is an exception from the Regalian Doctrine because it is considered private land even before the Spaniards came, “when the regalia doctrine was introduced into the

Philippines by colonizers, the colonizers did not intend to strip the natives of their ownership of lands already belonging to them.”

With regards to the Decree of June 25, 1880, it intended to correct the wrongful occupation by Filipinos of land belonging to the crown. There is no evidence that the possession of the petitioner is wrongful. Oh Cho v. Director of Lands G.R. No. 48321, 31 August 1946 FACTS: The opposition of the Director of Lands is based on the applicant's lack of title to the

lot, and on his disqualification, as alien, from acquiring lands of the public domain. The applicant, who is an alien, and his predecessors in interest have been in open, continuous, Page | 4

exclusive and notorious possession of the lot from 1880 to the filing of the application for registration on January 17, 1940. The applicant failed to show that he has title to the lot that may be confirmed under the Land Registration Act. He failed to show that he or any of his predecessors in interest had acquired the lot from the Government, either by purchase or by grant, under the laws, orders and decrees promulgated by the Spanish Government in the Philippines, or by possessory information under the Mortgage Law (section 19, Act 496).

ISSUE: Whether or not the applicant is entitled to a decree of registration thereof under the

provisions of the Public Land Act (C. A. No, 141)? RULING: No. Under the provisions of the Act invoked by the applicant, he is not entitled to a

decree of registration of the lot, because he is an alien disqualified from acquiring lands of the public domain (sections 48, 49, C. A. No. 141).The sale of the lot to the applicant should have been declared null and void.

Cruz v. Executive Secretary 347 SCRA 128

FACTS: RA 8371 was assailed as unconstitutional on the ground that it deprived the State of its ownership over lands of the public domain and the natural resources in them. RA 8371 defined what are ancestral domains and ancestral lands. HELD: The vote was 7-7 which meant that validity was upheld. The opinion defending constitutionality held the following: (1) ancestral domain and ancestral lands are not part of lands of the public domain. They are private and belong to indigenous people. Section 5 commands the state to protect the rights of indigenous people. Cariño v. Insular Government recognized native title held by Filipinos from time immemorial and excluded them from the coverage of  jura regalia . (2) The right of ownership granted does not include natural resources. The right to negotiate terms and conditions over natural resources covers only exploration to ensure environmental protection. It is not a grant of exploration rights. (3) The limited right of management refers to utilization as expressly allowed in Section 2, Article XII. (4) What is given is priority right, not exclusive right. It does not preclude the State from entering into coproduction, joint venture, or production sharing agreements with private entities. The opinion assailing the constitutionality of the law held the following: (1) the law amounts to an abdication of state authority over a significant area of the country’s patrimony; (2) it relinquishes full control of natural resources in favor of indigenous people; (3) the law contravenes the provision which says that all natural resources belong to the state.

C. ECCLESIASTICAL PRPOERTY Barlin v. Ramirez, 7 Phil. 41

FACTS: Barlin appointed Ramirez to administer Church property. When the former asked the latter to return the said property, the latter refused. He said that the property belongs to the State, and the same is granted to him by the State. ISSUE: WON Barlin should return the said property? HELD: Yes, he should return the said property. First, he is stopped by recognizing that the said property was only entrusted to him. Second the land belongs to the Roman Catholic Church. Since it belonged to the Roman Catholic Church, it was never public and therefore it was not included to the property ceded by Spain to the U.S. by virtue of the Treaty of Paris.

Roman Catholic Church v. Municipal of Tarlac, 9 Phil. 450 (Not in syllabus)

FACTS: Prior to the Revolution, the Church and cemetery is controlled and administered by the Roman Catholic Church. The said properties where destroyed during the revolution. On Page | 5

 January 10, 1903, by virtue of the circular, the Insular Government conveyed the land to the Independent Filipino Church for the purpose of administration in favor of the Municipality of Tarlac. The Roman Catholic Church seeks to get the property back. The respondent said that the RCC only administered the property but does not own it because it belongs to the State. ISSUE: WON the property involved belonged to the State. HELD: No, it belongs to the Church. As it was held in Barlin v. Ramirez, even though the property is administered by the Spanish government it belonged to the RCC and therefore private property. It could not have been part of the lands ceded to the U.S. by Spain. And because it is not part of the property ceded, it is not a property of the State and cannot by conveyed by it.

Friar lands were also an exception to the Regalian Doctrine because they are private lands at the time Spain ceded to the United States the Philippine Islands and by the time they were purchased by the government they were patrimonial property. III.

CLASSIFICATION and DISPOSITION of LANDS of the PUBLIC DOMAIN

The term “public lands” refer to such lands of the public domain as are subject to alienation and

disposal by the State in accordance with Commonwealth Act No. 141, of the Public Land Act. It does not include all lands of government ownership, but only so much of said lands as are thrown open to private appropriation and settlement. Accordingly, “government land” and “public land” are not synonymous terms; the first is more extensive and embraces not only the

second by also other lands of the government already reserved to public use or subject to private right. Montano v. The Insular Government Facts - Isabelo Montano presented a petition to the Court of Land Registration for the

inscription of a piece of land in the barrio of Libis, municipality of Caloocan, used as a fishery having a superficial area of 10,805 square meters, and bounded as set out in the petition; its value according to the last assessment being $505.05, United States currency. This petition was opposed by the Solicitor-General in behalf of the Director of Lands, and by the entity known as Obras Pias de la Sagrada Mitra, the former on the ground that the land in question belonged to the Government of the United States, and the latter, that it was the absolute owner of all the dry land along the eastern boundary of the said fishery. The Court of Land Registration in its decision of December 1, 1906, dismissed the said oppositions without costs and decreed, after a general entry by default, the adjudication and registration of the property described in the petition, in favor of Isabelo Montano y Marcial. From this decision only counsel for the Director of Public Lands appealed to this court. And precisely Isabelo Montano sought title thereon on the strength of 10 years' occupation pursuant to paragraph 6, section 5 of Act 926 of the Philippine Commission Issue: Whether the lands used as a fishery , for the growth of nipa, and as salt deposits, inland

some distance from the sea, and asserted, though not clearly proved to be overflowed at high tide could be registered as private property, and whether it (land in question) can be acquired by Montano. Held: Yes.

Accordingly, "government land" and "public domain" are not synonymous items. The first includes not only the second, but also other lands of the Government already reserved or devoted to public use or subject to private right. In other words, the Government owns real estate which is part of the "public lands" and other real estate which is not part thereof. Government property was of two kinds — first, that of public use or service, said to be of public ownership, and second, that of having a private character or use. (Civil Code, arts. 339 and 340.) Lands of the first class, while they retain their public character are inalienable. Those of the second are not. Therefore, there is much real property belonging to the Government which is not affected by statutes for the settlement, prescription or sale of public lands. Examples in point are properties occupied by public buildings or devoted to municipal or other governmental uses. It is settled that the general legislation of Congress in respect to public lands does not extend to tide lands. It provided that the scrip might be located on the unoccupied and unappropriated Page | 6

public lands. As said in Newhall vs. Sanger(92 U.S. 761, 763.) A marshland which is inundated by the rise of tides belong to the State and is not susceptible to appropriation by occupation, has no application in the present case inasmuch as in said case the land subject matter of the litigation was not yet titled. Lands under tide waters are incapable of cultivation or improvement in the manner of lands above high-water mark. They are of great value to the public for the purposes of commerce, navigation, and fishery. Their improvement by individuals, when permitted, is incidental or subordinate to the public use and right. Therefore the title and the control of them are vested in the sovereign for the benefit of the whole people. But due to uncertain and somewhat unsatisfactory condition of the law the custom had grown up of converting manglares and nipa lands into fisheries which became common feature of settlements along the coast and at the time of the change of sovereignty constituted one of the most productive industries of the Islands, the abrogation of which would destroy vested interests and prove a public disaster. In our opinion it was the object of Congress not to work such a result but, on the contrary, in furtherance of the purposes of the treaty of Paris, to recognize and safeguard such property. Therefore, the judgment of the Court of Land Registration is affirmed, without costs.  A. Classification of Lands of the Public Domain 1. GENERAL C LASSIFICATION OF L ANDS OF THE PUBLIC DOMAIN 

As already set forth above, Section 3, Article XII of the 1987 Constitution classifies lands of the public domain as (1) agricultural, (2) forest or timber, (3) mineral and (4) national parks. The classification is descriptive of the legal nature of the land and not of what it looks like. Furthermore, under Section 2, Article XII, alienable lands of the public domain under the Constitution are limited only to agricultural lands. This is reflected in Section 6 of Commonwealth Act No. 141 of the Public Land Act. While the Public Land Act generally deals only with alienable lands of the public domain,1 it nonetheless provides the following specific provision: SECTION 6. The President, upon the recommendation of the Secretary of Agriculture and Commerce, shall from time to time classify the lands of the public domain into — (a) Alienable or disposable, (b) Timber, and (c) Mineral lands, and may at any time and in a like manner transfer such lands from one class to another, for the purposes of their administration and disposition.

Notably, the power to classify lands of the public domain is vested in the President. In Director of Lands v. Court of Appeals,2 the Supreme Court, applying the foregoing provision, ruled that “the classification of public land is an exclusive prerogative of the Executive Department of the Government and not of the Courts. In the absence of such classification, the lands remain as unclassified land until it is released therefrom and rendered open to disposition. This should be so under time honored Constitutional precepts. This is also in consonance with the Regalian Doctrine that all lands of the public domain belong to the State, and that the State is the source of any asserted right to ownership in the land and charged with the conservation of such patrimony.” Such classifications, however, must be categorical: that is, land is either completely agricultural or completely mineral or completely forest or park. 3 A land cannot have a mixed classification. In Republic v. Court of Appeals,4 “the Court feels that the rights over the land are indivisible and that the land itself cannot be half agricultural and half mineral. The classification must be categorical: the land must be either completely mineral or completely agricultural.”

1 2 3 4

See Section 2 of the Public Land Act.

129 SCRA 689 BERNAS, COMMENTARY supra note 13, at 1145. 160 SCRA 228 (1988). Page | 7

Furthermore, the Court ruled in Director of Lands v. Judge Aquino ,5 that the classification of land does not change when the nature of the land changes. A positive act of the executive is nonetheless needed. Anyone who claims that the classification has been changed must be able to show the positive act of the President indicating such positive act.

Director of Lands v. Aquino, 192 SCRA 296

FACTS: Abra industrial Corporation sought to register a “limestone -rich 70 hectar land in Bucay, Abra 66 hectars of which allegedly belongs to the Central Cordillera Forest Reserve. The lower court gave due course to the application. It ruled that although part of the land is within the forest reserve the Bureau of Forestry “offered no objection to exclude the same area from the forest reserve.” The Director Lands contends such ruling as incorrect.

ISSUE: WON the Lower Court is correct in granting the application for registration. HELD: No, The lack of objection on the part of the Bureau of Forestry is of no moment because the classification of lands is privilege given only to the President. Without any positive act from him, a land cannot be classified nor re-classified.In the present case, there was no positive act whatsoever from the executive department classifying such land an alienable or disposable. Therefore, the application for registration must be denied because the land involved cannot be alienated because it is Forest Land.

Director of Lands v. Court of Appeals, 211 SCRA 869

FACTS: Respondents tried to register a certain parcel of land. They have used the said land for raising livestock for many years. The government opposed stating the land is classified as a forest land. ISSUE: WON the registration proper. HELD: No. The power to classify lands of public domain resides in the executive department. And if there is proof the executive department that such land is classified as a forest land therefore the burden of proof is upon the applicant to show that the involved land is already classified as alienable. In the present case, the applicant failed to show such proof. The Court also held that the word “timber” land in the Public Land Act is the same as “forest” land in the

Constitution. And even if the land does not look like a forest it is still forest land as long as it is classified as such.

It should also be stressed, however, that, by virtue of the Regalian Doctrine, the power of the executive to classify lands of the public domain is only a delegated power by the Philippine legislature. Thus, under Section 6 of the Public Land Act, both the President and Congress are empowered to declare what public lands are open to disposition or concession under this Act. In addition to the power to classify lands of the public domain into alienable or disposable, timer, mineral lands and, through the Nipas Act, national parks, the President also has the power to declare otherwise alienable or disposable lands as reservations, for town sites and for other public and semipublic purposes.6 The effect of such reservation is that such land, while alienable, cannot be transferred to public individuals, and must be held by the State, either through the national government or a corresponding local government unit.

Cases:

5 6

G.R. No. 31688, December 17, 1990. See Title V of the Public Land Act. Page | 8

Director of Lands v. Court of Appeals, G.R. No. 58867, 1984

FACTS: In their application for registration filed on May 10, 1976, private respondents (Applicants, for brevity) claimed that they are the co-owners in fee simple of the land applied for partly through inheritance in 1918 and partly by purchase on May 2, 1958; that it is not within any forest zone or military reservation; and that the same is assessed for taxation purposes in their names. The Republic of the Philippines, represented by the Director of the Bureau of Forest Development opposed the application on the principal ground that the land applied for is within the unclassified region of Obando, Bulacan, per BF Map LC No. 637 dated March 1, 1927; and that areas within the unclassified region are denominated as forest lands and do not form part of the disposable and alienable portion of the public domain. After hearing, the Trial Court ordered registration of the subject land in favor of the Applicants. This was affirmed on appeal by respondent Appellate Court. ISSUE: Whether or not Courts can reclassify the subject public land. HELD: No. The classification of public lands is an exclusive prerogative of the Executive Department of the Government and not of the Courts. In the absence of such classification, the land remains as unclassified land until it is released therefrom and rendered open to disposition. This should be so under time-honored Constitutional precepts. This is also in consonance with the Regalian doctrine that all lands of the public domain belong to the State, and that the State is the source of any asserted right to ownership in land and charged with the conservation of such patrimony.

Secretary of DENR v. Yap, 2008 FACTS: On November 10, 1978 , then President Marcos issued Proc. No. 1801 declaring Boracay Island, among other islands, caves and peninsulas in the Philippines, as tourist zones and marine reserves under the administration of the Philippine Tourism Authority (PTA). President Marcos later approved the issuance of PTA Circular 3-82 dated September 3, 1982, to

implement Proclamation No. 1801. Claiming that Proclamation No. 1801 and PTA Circular No 3-82 precluded them from filing an application for judicial confirmation of imperfect title or survey of land for titling purposes, respondents-claimants Mayor . Yap, Jr., and others filed a petition for declaratory relief with the RTC in Kalibo, Aklan In their petition, respondents-claimants alleged that Proc. No. 1801 and PTA Circular No. 3-82 raised doubts on their right to secure titles over their occupied lands. They declared that they themselves, or through their predecessors-in-interest, had been in open, continuous, exclusive, and notorious possession and occupation in Boracay since June 12, 1945, or earlier since time immemorial. They declared their lands for tax purposes and paid realty taxes on them. Respondents-claimants posited that Proclamation No. 1801 and its implementing Circular did not place Boracay beyond the commerce of man. Since the Island was classified as a tourist zone, it was susceptible of private ownership. Under Section 48(b) of the Public Land Act, they had the right to have the lots registered in their names through judicial confirmation of imperfect titles. The Republic, through the OSG, opposed the petition for declaratory relief. The OSG countered that Boracay Island was an unclassified land of the public domain. It formed part of the mass of lands classified as “public forest,” which was not available for disposition pursuant

to Section 3(a) of the Revised Forestry Code, as amended. The OSG maintained that respondents-claimants’ reliance on PD No. 1801 and PTA Circu lar No. 3-82 was misplaced. Their right to judicial confirmation of title was governed by Public Land Act and Revised Forestry Code, as amended. Since Boracay Island had not been classified as alienable and disposable, whatever possession they had cannot ripen into ownership.

Page | 9

On July 14, 1999, the RTC rendered a decision in favor of respondents-claimants, declaring that, “PD 1810 and PTA Circular No. 3 -82 Revised Forestry Code,as amended. The OSG moved for reconsideration but its motion was denied. The Republic then appealed to the CA. On In 2004, the appellate court affirmed in toto the RTC decision. Again, the OSG sought reconsideration but it was similarly denied. Hence, the present petition under Rule 45. On May 22, 2006, during the pendency the petition in the trial court, President Gloria Macapagal-Arroyo issued Proclamation No. 1064 classifying Boracay Island partly reserved forest land (protection purposes) and partly agricultural land (alienable and disposable). On August 10, 2006, petitioners-claimants Sacay,and other landowners in Boracay filed with this Court an original petition for prohibition, mandamus, and nullification of Proclamation No. 1064. They allege that the Proclamation infringed on their “prior vested rights” over portions of Boracay. They have been in continued possession of their respective lots in Boracay since time immemorial. On November 21, 2006, this Court ordered the consolidation of the two petitions ISSUE: the main issue is whether private claimants have a right to secure titles over their

occupied portions in Boracay. HELD: petitions DENIED. The CA decision is reversed.

 Except for lands already covered by existing titles, Boracay was an unclassified land of the  public domain prior to Proclamation No. 1064. Such unclassified lands are considered public  forest under PD No. 705.

PD No. 705 issued by President Marcos categorized all unclassified lands of the public domain as public forest. Section 3(a) of PD No. 705 defines a public forest as “a mass of lands of the public domain which has not been the subject of the present system of classificatio n for the determination of which lands are needed for forest purpose and which are not.” Applying PD No. 705, all unclassified lands, including those in Boracay Island, are ipso facto considered public forests. PD No. 705, however, respects titles already existing prior to its effectivity. The 1935 Constitution classified lands of the public domain into agricultural, forest or timber, such classification modified by the 1973 Constitution. The 1987 Constitution reverted to the 1935 Constitution classification with one addition: national parks. Of these, only agricultural lands may be alienated. Prior to Proclamation No. 1064 of May 22, 2006, Boracay Island had never been expressly and administratively classified under any of these grand divisions. Boracay was an unclassified land of the public domain.  A positive act declaring land as alienable and disposable is required . In keeping with the presumption of State ownership, the Court has time and again emphasized that there must be a positive act of the government, such as a presidential proclamation or an executive order;

an administrative action; investigation reports of Bureau of Lands investigators; and a legislative act or a statute. The applicant may also secure a certification from the government that the land claimed to have been possessed for the required number of years is alienable and disposable. The burden of proof in overcoming such presumption is on the person applying for registration (or claiming ownership), who must prove that the land subject of the application is alienable or disposable. In the case at bar, no such proclamation, executive order, administrative action, report, statute, or certification was presented to the Court. The records are bereft of evidence showing that, prior to 2006, the portions of Boracay occupied by private claimants were subject of a government proclamation that the land is alienable and disposable. Matters of land classification or reclassification cannot be assumed. They call for proof. Proc. No. 1801 cannot be deemed the positive act needed to classify Boracay Island as alienable and disposable land. If President Marcos intended to classify the island as alienable and disposable or forest, or both, he would have identified the specific limits of each, as President Arroyo did in Proclamation No. 1064. This was not done in Proclamation No. 1801. Page | 10

NOTES:

1. Private claimants’ reliance on  Ankron and De Aldecoa is misplaced.  Ankron and De  Aldecoa were decided at a time when the President of the Philippines had no power to classify lands of the public domain into mineral, timber, and agricultural. At that time, the courts were free to make corresponding classifications in justiciable cases, or were vested with implicit power to do so, depending upon the preponderance of the evidence. Act No. 2874, promulgated in 1919 and reproduced in Section 6 of Public Land Act, gave the Executive Department, through the President, the exclusive prerogative to classify or reclassify public lands into alienable or disposable, mineral or forest. Since then, courts no longer had the authority, whether express or implied, to determine the classification of lands of the public domain. 2. Each case must be decided upon the proof in that particular case , having regard for its present or future value for one or the other purposes . We believe, however, considering the fact that it is a matter of public knowledge that a majority of the lands in the Philippine Islands are agricultural lands that the courts have a right to presume, in the absence of evidence to the contrary, that in each case the lands are agricultural lands until the contrary is shown.  Whatever the land involved in a particular land registration case is forestry or mineral land must, therefore, be a matter of proof. Its superior value for one purpose or the other is a question of fact to be settled by the proof in each particular case

Forests, in the context of both the Public Land Act and the Constitution classifying lands of the public domain into “ agricultural, forest or timber, mineral lands, and national parks ,” do not

necessarily refer to large tracts of wooded land or expanses covered by dense growths of trees and under brushes. The discussion in Heirs of Amunategui v. Director of Forestry is particularly instructive: A forested area classified as forest land of the public domain does not lose such classification simply because loggers or settlers may have stripped it of its forest cover. Parcels of land classified as forest land may actually be covered with grass or planted to crops by kaingin cultivators or other farmers. “Forest lands” do not have to be on mou ntains or in out of the way places. Swampy areas covered by mangrove trees, nipa palms, and other trees growing in brackish or sea water may also be classified as forest land. The classification is descriptive of its legal nature or status and does not have to be descriptive of what the land actually looks like. Unless and until the land classified as “forest” is released in an official

proclamation to that effect so that it may form part of the disposable agricultural lands of the public domain, the rules on confirmation of imperfect title do not apply. There is a big difference between “forest” as defined in a dictionary and “forest or timber land”

as a classification of lands of the public domain as appearing in our statutes. One is descriptive of what appears on the land while the other is a legal status, a classification for legal purposes. At any rate, the Court is tasked to determine the legal status of Boracay Island, and not look into its physical layout. Hence, even if its forest cover has been replaced by beach resorts, restaurants and other commercial establishments, it has not been automatically converted from public forest to alienable agricultural land. 3. All is not lost, however, for private claimants. While they may not be eligible to apply for  judicial confirmation of imperfect title under Section 48(b) of CA No. 141, as amended, this does not denote their automatic ouster from the residential, commercial, and other areas they possess now classified as agricultural. Neither will this mean the loss of their substantial investments on their occupied alienable lands. Lack of title does not necessarily mean lack of right to possess. For one thing, those with lawful possession may claim good faith as builders of improvements. They can take steps to preserve or protect their possession. For another, they may look into other modes of applying for original registration of title, such as by homestead or sales patent, subject to the conditions imposed by law. More realistically, Congress may enact a law to entitle private claimants to acquire title to their occupied lots or to exempt them from certain requirements under the present land laws. There is one such bill now pending in the House of Representatives. 2.  MANNER AND EFFECT OF CLASSIFICATION  Republic v. Court of Appeals and De la Rosa , 160 SCRA 228 (1988)

Page | 11

Doctrine: The owner of a piece of land has rights not only to its surface but also to everything

underneath and the airspace above it up to a reasonable height. The rights over the land are indivisible and the land itself cannot be half agricultural and half mineral. The classification must be categorical; the land must be either completely mineral or completely agricultural. Facts: These cases arose from the application for registration of a parcel of land filed on February

11, 1965, by Jose de la Rosa on his own behalf and on behalf of his three children, Victoria, Benjamin and Eduardo. The land, situated in Tuding, Itogon, Benguet Province, was divided into 9 lots and covered by plan Psu-225009. According to the application, Lots 1-5 were sold to Jose de la Rosa and Lots 6-9 to his children by Mamaya Balbalio and Jaime Alberto, respectively, in 1964. The application was separately opposed by Benguet Consolidated, Inc. as to Lots 1-5, Atok Big Wedge Corporation, as to Portions of Lots 1-5 and all of Lots 6-9, and by the Republic of the Philippines, through the Bureau of Forestry Development, as to lots 1-9. In support of the application, both Balbalio and Alberto testified that they had acquired the subject land by virtue of prescription Balbalio claimed to have received Lots 1-5 from her father shortly after the Liberation. Benguet opposed on the ground that the June Bug mineral claim covering Lots 1-5 was sold to it on September 22, 1934, by the successors-in-interest of James Kelly, who located the claim in September 1909 and recorded it on October 14, 1909. From the date of its purchase, Benguet had been in actual, continuous and exclusive possession of the land in concept of owner, as evidenced by its construction of adits, its affidavits of annual assessment, its geological mappings, geological samplings and trench side cuts, and its payment of taxes on the land. For its part, Atok alleged that a portion of Lots 1-5 and all of Lots 6-9 were covered by the Emma and Fredia mineral claims located by Harrison and Reynolds on December 25, 1930, and recorded on January 2, 1931, in the office of the mining recorder of Baguio. These claims were purchased from these locators on November 2, 1931, by Atok, which has since then been in open, continuous and exclusive possession of the said lots as evidenced by its annual assessment work on the claims, such as the boring of tunnels, and its payment of annual taxes thereon. The Bureau of Forestry Development also interposed its objection, arguing that the land sought to be registered was covered by the Central Cordillera Forest Reserve under Proclamation No. 217 dated February 16, 1929. Moreover, by reason of its nature, it was not subject to alienation under the Constitutions of 1935 and 1973. The trial court denied the application, holding that the applicants had failed to prove their claim of possession and ownership of the land sought to be registered. The applicants appealed to the respondent court, which reversed the trial court and recognized the claims of the applicant, but subject to the rights of Benguet and Atok respecting their mining claims. In other words, the Court of Appeals affirmed the surface rights of the de la Rosas over the land while at the same time reserving the sub-surface rights of Benguet and Atok by virtue of their mining claims. Both Benguet and Atok have appealed to this Court, invoking their superior right of ownership. Issue: Whether respondent court’s decision, i.e. “the surface rights of the de la Rosas over the

land while at the same time reserving the sub-surface rights of Benguet and Atok by virtue of their mining claim,” is correct. Held: No. Our holding is that Benguet and Atok have exclusive rights to the property in question

by virtue of their respective mining claims which they validly acquired before the Constitution of 1935 prohibited the alienation of all lands of the public domain except agricultural lands, subject to vested rights existing at the time of its adoption. The land was not and could not have been transferred to the private respondents by virtue of acquisitive prescription, nor could its use be shared simultaneously by them and the mining companies for agricultural and mineral purposes. It is true that the subject property was considered forest land and included in the Central Cordillera Forest Reserve, but this did not impair the rights already vested in Benguet and Atok at that time. Such rights were not affected either by the stricture in the Commonwealth Constitution against the alienation of all lands of the public domain except those agricultural in Page | 12

nature for this was made subject to existing rights. The perfection of the mining claim converted the property to mineral land and under the laws then in force removed it from the public domain. By such act, the locators acquired exclusive rights over the land, against even the government, without need of any further act such as the purchase of the land or the obtention of a patent over it. As the land had become the private property of the locators, they had the right to transfer the same, as they did, to Benguet and Atok. The Court of Appeals justified this by saying there is “no conflict of interest” between the owners of the surface rights and the owners of the

sub-surface rights. This is rather doctrine, for it is a well-known principle that the owner of piece of land has rights not only to its surface but also to everything underneath and the airspace above it up to a reasonable height. Under the aforesaid ruling, the land is classified as mineral underneath and agricultural on the surface, subject to separate claims of title. This is also difficult to understand, especially in its practical application. The Court feels that the rights over the land are indivisible and that the land itself cannot be half agricultural and half mineral. The classification must be categorical; the land must be either completely mineral or completely agricultural. In the instant case, as already observed, the land which was originally classified as forest land ceased to be so and became mineral — and completely mineral — once the mining claims were perfected. As long as mining operations were being undertaken thereon, or underneath, it did not cease to be so and become agricultural, even if only partly so, because it was enclosed with a fence and was cultivated by those who were unlawfully occupying the surface. This is an application of the Regalian doctrine which, as its name implies, is intended for the benefit of the State, not of private persons. The rule simply reserves to the State all minerals that may be found in public and even private land devoted to “agricultural, industrial, commercial, residential or (for) any purpose other than mining.” Thus, if a person is the owner of agricultural

land in which minerals are discovered, his ownership of such land does not give him the right to extract or utilize the said minerals without the permission of the State to which such minerals belong. The flaw in the reasoning of the respondent court is in supposing that the rights over the land could be used for both mining and non-mining purposes simultaneously. The correct interpretation is that once minerals are discovered in the land, whatever the use to which it is being devoted at the time, such use may be discontinued by the State to enable it to extract the minerals therein in the exercise of its sovereign prerogative. The land is thus converted to mineral land and may not be used by any private party, including the registered owner thereof, for any other purpose that will impede the mining operations to be undertaken therein, For the loss sustained by such owner, he is of course entitled to just compensation under the Mining Laws or in appropriate expropriation proceedings.

Pangkatipuran v. Court of Appeals, 379 SCRA 621 (Regalian Doctrine case )

DOCTRINE: Under the Regalian doctrine, all lands of the public domain belong to the State, and the State is the source of any asserted right to ownership in land and charged with the conservation of such patrimony. This same doctrine also states that all lands not otherwise appearing to be clearly within private ownership are presumed to belong to the State. To overcome such presumption, incontrovertible evidence must be shown by the applicant that the land subject of the application is alienable or disposable. FACTS: Petitioner filed with the Court of First Instance an application for judicial confirmation and registration of title to certain lots. The Court of First Instance promulgated a decision confirming petitioners’ title to the property. The Official Certificate Title was issue d in the name of the petitioners. Republic of the Philippines filed with the Intermediate Appellate Court an action to declare the proceedings and titles derived therefrom as null and void, to direct the register of deeds to annul said certificates of title, and to confirm the subject land as part of the public domain. The Court of Appeals set aside the June 15, 1967 decision of the court a quo and ordered that the subject lot be reverted back to the public domain. Page | 13

HELD: The petition is bereft of merit. Unless public land is shown to have been reclassified or alienated to a private person by the State, it remains part of the inalienable public domain. Occupation thereof in the concept of owner, no matter how long, cannot ripen into ownership and be registered as a title. Evidence on record showed that at the time of filing of the application for land registration and issuance of the certificate of title over the disputed land in the name of petitioners, the same was timberland and formed part of the public domain. Based on a wrong concept of what is forest land, the court a quo found registrable title in favor of petitioners based on the Republic’s failure

to show that the land is more valuable as forest land than for agricultural purposes. There was no evidence showing that the land has been reclassified as disposable or alienable. Before any land may be declassified from the forest group and converted into alienable or disposable land for agricultural or other purposes, there must be a positive act from the government. Even rules on the confirmation of imperfect titles do not apply unless and until the land classified as forest land is released in an official proclamation to that effect so that it may form part of the disposable agricultural lands of the public domain. Declassification of forest land is an express and positive act of Government. It cannot be presumed. Neither should it be ignored nor deemed waived. Since the land in question is unregistrable, the land registration court did not acquire jurisdiction over the same. Any proceedings had or judgment rendered therein is void and is not entitled to the respect accorded to a valid judgment. (Petitioners’ contention that the Republic is now barred from questioning the validity of the

certificate of title issued to them considering that it took the government almost eighteen (18) years to assail the same is erroneous. It is a basic precept that prescription does not run against the State. The lengthy occupation of the disputed land by petitioners cannot be counted in their favor, as it remained part of the patrimonial property of the State, which property, as stated earlier, is inalienable and indisposable.) 3.  ADMINISTRATION OF PUBLIC LAND

Commonwealth Act No. 141 Ҥ3. The Secretary of Agriculture and Commerce shall be the executive officer charged with

carrying out the provisions of this Act through the Director of Lands, who shall act under his immediate control. §5. The Director of Lands, with the approval of the Secretary of Agriculture and Commerce shall prepare and issue such forms, instructions, rules, and regulations consistent with this Act, as may be necessary and proper to carry into effect the provisions thereof and for the conduct of proceedings arising under such provisions. §4. Subject to said control, the Director of Lands (as delegated by the President – E.O 292) shall have direct executive control of the survey, classification, lease, sale or any other form of concession or disposition and management of the lands of the public domain, and his decisions as to questions of fact shall be conclusive when approved by the Secretary of Agriculture and Commerce. B. T  HE PUBLIC L AND ACT AND THE C LASSIFICATION OF  A LIENABLE L ANDS OF THE PUBLIC DOMAIN  1. Brief history of public land laws in the Philippines

The law on the classification and disposition of lands of the public domain traces its roots to the Regalian Doctrine, through which “all lands that were not acquired from the Governmen t, either by purchase or by grant, belong to the public domain. ” This was decreed under the Novisima Recopliacion de Leyes delas Indias.7  The Novisima Recopliacion was followed by the Ley Hipotecaria or the Mortgage Law of 1893 which provided for the systematic registration of titles and deeds as well

7

Collado v. Court of Appeals, 390 SCRA 343 [2002], citing Chavez v. Public Estates Authority, 384 SCRA 152 [2002]. Page | 14

as possessory claims.8 The Royal Decree of 1894 or the Maura Law, which was effective on 13 February 1894, partly amended the Spanish Mortgage Law and the Novisima Recopliacion by establishing possessory information as the method of legalizing possession of vacant Crown land. Through these royal issuances, therefore, private ownership of land under the Spanish regime, which was based on royal concessions, took the following forms: (a) titulo real or royal grant; (b) concesion especial or special grant; (c) composicion con el estado or adjustment title; (d) titulo de compra or title by purchase; and (e) informacion posesoria or possessory information title.9 With the defeat of the Spain during the Spanish – American War, sovereignty over the Philippine islands, including lands of the public domain owned by the King, was transferred to the United States through the Treaty of Paris of 1898. The Philippine Bill of 1902,10 thereafter classified lands of the public domain as agricultural, mineral and timber or forest lands.11 The act provided for, among others, the disposal of mineral lands by means of absolute grant (freehold system) and by lease (leasehold system). It also provided a definition, by exclusion, of “agricultural public lands,” which the Supreme Court, in the case of  Mapa vs. Insular Government,12 interpreted to mean: “those public lands acquired from Spain which are not timber or mineral lands. . . .”

The first Public Land Act, Act No. 926, was then issued by the American colonial government in pursuance of the Philippine Bill of 1902. The Act introduced the homestead system and made provisions for judicial and administrative confirmation of imperfect titles and for the sale or lease of public lands. It also permitted corporations regardless of the nationality of persons owning the controlling stock to lease or purchase lands of the public domain. Act No. 926 was replaced by Act No. 2874 on 7 October 1903. This new, more comprehensive law limited the exploitation of agricultural lands to Filipinos and Americans and citizens of other countries which gave Filipinos the same privileges. With the inauguration of the Philippine Commonwealth and the 1935 Constitution which adopted and applied the Regalian Doctrine to our republican system, Congress passed the present Public Land Act on 1 December 1936 in order to make all pre-existing public land laws conform to the nationalistic provisions of the 1935 Constitution.13 Through the Public Land Act, all pre-existing laws relative to the disposition of public lands were complied in a single instrument. 2. Coverage

Section 2 of the Public Land Act provides for the law’s application and coverage:

SECTION 2. The provisions of this Act shall apply to the lands of the public domain; but timber and mineral lands shall be governed by special laws and nothing in this Act provided shall be understood or construed to change or modify the administration and disposition of the lands commonly called “friar lands” and those which, being privately owned, have reverted to or become the property of the Commonwealth of the Philippines, which administration and disposition shall be governed by the laws at present in force or which may hereafter be enacted.

Excepted from the coverage of the law are the disposition, use and administration of timber and mineral lands, which, as earlier mentioned, are today respectively governed by the Revised Forestry Code and the Mining Act of 1995. Also excepted are the so-called “Friar Lands,” which are lands acquired by the government during the Taft Administration in 1903 from religious corporations or orders, for administration, temporary leasing, and eventual sale to actual occupants of the land. By express provision of Act 8 9

Collado v. Court of Appeals, supra. Noblejas, A.H. and Noblejas, E.H., Registration of Land Titles and Deeds 5 – 11, cited in Secretary of the DENR, et al. vs. Yap, et al., G.R. No. 167707 & 173775, 8 October 2008.

10 Entitled, “An Act Temporarily to Provide for the Administration of the Affairs of Civil Government in the Philippine Islands, and for Other Purposes,” issued on July 1, 1902, 11 Director of Forestry v. Villareal, 170 SCRA 598, 601 [1989]. 12 10 Phil. 175 (1908). 13 PEÑA, NATURAL RESOURCES, supra note 30 at 18. Page | 15

No. 1120, friar lands are not considered public lands in the sense in which this word is used in the Public Land Act, and are, instead, considered private and patrimonial property of the State.14 Thus, their acquisition is not governed by the provisions of the Public Land Act, but instead by Act No. 1120, and later, by Commonwealth Act No. 32, 15 as amended by Commonwealth Act No. 316. Also excluded from the coverage of the Public Land Act are lands within big landed estates acquired by the government pursuant to Section 4, Article XIII of the 1935 Constitution 16 which have been sold to tenants under provisions of Commonwealth Act Nos. 20 17 and 539.18 These are also considered private lands of the government. 19 It should be underscored that when ownership over public lands are transferred under any of the methods provided by the Public Land Act ( i.e. homestead, sale, judicial confirmation of imperfect title, etc.), these lands are no longer to be governed by the provisions of the said Act, and are considered private lands.20 This notwithstanding, the Supreme Court ruled that even if the lands so acquired have become private property, these lands still remain subject to the limitations and restrictions imposed by the Public Land Act on alienation, mortgage, or repurchase.21 3. Classification of Public or Alienable Lands of the Public Domain

As we have seen previously, only agricultural lands of the public domain can be transferred or disposed of to qualified persons. Section 8 of the Public Land Act defines such lands as those which satisfy the following requirements: (a) The land must have been declared open to disposition or concession; (b) The land must have been officially delimited and classified, and, when practicable, surveyed;  provided that, the President may, for reasons of public interest, declare lands of the public domain open to disposition before the same have had their boundaries established, or been surveyed; and, (c) The land must not have been reserved for public or quasi-public uses, nor appropriated by the Government, nor in any manner become private property.

These public agricultural lands are further classified under Section 9 of the P ublic Land Act as: (a) Agricultural, (b) Residential, commercial, industrial, or for similar productive purposes; (c) Educational, charitable, or other similar purposes; and 14 Jacinto vs. Director of Lands, 49 Phil. 853 [1926]. 15 Entitled, “An Act providing for the subdivision and sale of all the portions of Friar Lands Estates remaining undisposed of,” issued on 15 September 1936.

16 Section 4. The Congress may authorize, upon payment of just compensation, the expropriation of lands to be subdivided into small lots and conveyed at cost to individuals. 17 Entilted, “An Act authorizing the President of the Philippines to order the institution of expropriation proceedings or to enter into negotiations fo rthe purpose of acquiring portions of large landed estates used as homesites and reselling them at cost to their bona fide occupants, appropriting funds therefor, and for other purposes,” issued on 11 July 1936. 18 Entitled, “AN ACT AUTHORIZING THE PRESIDENT O F THE PHILIPPINES TO ACQUIRE PRIVATE LANDS

FOR RESALE IN SMALL LOTS; PROVIDING FOR THE CREATION OF AN AGENCY TO CARRY OUT THE PURPOSES OF THIS ACT; AND SETTING ASIDE FUNDS AND AUTHORIZING THE ISSUANCE OF BONDS FOR THE PAYMENT OF SAID LANDS,” issued on 26 M ay 1940. 19 Commonwealth Act No. 20 authorizes the President to order the institution of expropriation proceedings or to enter into negotiations for the purpose of acquiring portions of large landed estates which are now used as home sites and reselling them at cost to their bona fide occupants. Meanwhile, Commonwealth Act No. 539 allows the President to acquire private lands or any interest therein, through purchase or expropriation, and to subdivide them into home lots or small farms for resale at reasonable prices to their bona fide tenants or occupants or to private individuals who will work the lands themselves and who are qualified to acquire and own lands in the Philippines. 20 Central Capiz v. Ramirez, GR No. 40399, 40 Phil. 883 (1920). 21 Francisco, et al. vs. Certeza, Sr., et al., 3 SCRA 565 [1961]. Page | 16

(d) Reservations for town sites and for public and quasi-public uses.

In turn, alienable lands classified as residential, commercial and industrial lands under Section 9 of the Public Land Act are further classified as: (a) Lands reclaimed by the government by dredging, filling, or other means, (b) Foreshore; (c) Marshy lands or lands covered with water bordering upon the shores or bank of navigable lakes or rivers; and, (d) Land not included in any of the foregoing classes.

It should be noted that while Section 9 of the Public Land Act classifies agricultural land separate from residential, commercial, educational, reservations, etc., all lands enumerated under the said provision all correspond to agricultural land as understood under Section 3, Article XII of the Constitution as open to alienation or disposition. The classification under Section 9, therefore, is for purposes of administration and disposition , or according to the purpose to which said lands are especially adapted. Notwithstanding this classification, all of said lands are essentially agricultural lands which may be alienated. C.  M ODES OF  D ISPOSITION OF  PUBLIC L ANDS OF ALIENABLE LANDS OF PUBLIC DOMAIN 

As we have learned under the Regalian Doctrine, no public land can be acquired by private persons without any grant, express or implied from the government. In other words, it is indispensable that there be a showing of a title form the state. One claiming “private rights” must prove that he has complied with the Public Land Act which prescribes the substantive as well as the procedural requirements for acquisition of public lands.22 Furthermore, only those lands shall be declared open to disposition or concession which have been officially delimited and classified and, when practicable, surveyed, and which have not been reserved for public or quasi-public uses, nor appropriated by the government, nor in any manner become private property, nor those on which a private right authorized and recognized by the Act or any valid law may be claimed or which, having been reserved or appropriated have ceased to be so. In the absence of such classification, the land remains as unclassified land until it is released therefrom and rendered open to disposition.23 In Menguito v. Republic,24 it was held that unless public land is shown to have been reclassified or alienated to a private person by the State, it remains part of the inalienable public domain. Indeed, occupation thereof in the concept of owner, no matter how long, cannot ripen into ownership and be registered as a title. Menguito vs Republic, 2000

Facts: Petition for Review under Rule 45 of the Rules of Court assailing the CA decision reversing the RTC order granting Menguito’s application for registration of land.

Menguito et al applied for registration of title alleging they are owners in fee simple of eleven (11) parcels of land situated in the Barrio of Ususan, Municipality of Taguig, Metro Manila. They maintained they acquired the land by inheritance and have been paying taxes for the said land and no other persons have any estate or interest therein, legal or equitable, in possession, remainder, reversion or expectancy. The Republic opposed the application alleging that neither the applicant nor his predecessors-ininterest have been in open, continuous, exclusive and notorious possession and occupation of the land in question since June 12, 1945 or prior thereto and the muniments of title and tax payment receipts of applicant, if any, attached to or alleged in the application, do not constitute competent and sufficient evidence of a bona fide acquisition of the lands applied for or his open, continuous, exclusive and notorious possession and occupation thereof in the concept of owner, since June 12, 1945, or prior thereto. Said muniments of title do not appear to be genuine and indicate the pretended possession of applicant to be of recent vintage. Further, the parcel applied is part of

22 AGCAOILI, NATURAL RESOURCES supra note 15, at 19. 23 AGCAOILI, NATURAL RESOURCES supra note 15, at 20. 24 348 SCRA 128 (2000). Page | 17

the public domain belonging to the Republic of the Philippines not subject to private appropriation. ISSUE: WON Menguito has title to the disputed land HELD: Petition is DENIED and the assailed Decision AFFIRMED. Petitioners were duty-bound to prove two legal requirements: (1) the land applied for was alienable and disposable; and (2) the applicants and their predecessors-in-interest had occupied and possessed the land openly, continuously, exclusively, and adversely since June 12, 1945. The records show that petitioners failed to establish these two requisites. To prove that the land in question formed part of the alienable and disposable lands of the public domain, petitioners cite a surveyor-geodetic engineer’s notation in Exhibit “E” indicating that the survey was inside alienable and disposable land. Such notation does not constitute a positive government act validly changing the classification of the land in question. Verily, a mere surveyor has no authority to reclassify lands of the public domain. Such proof is insufficient as unless public land is shown to have been reclassified or alienated to a private person by the State, it remains part of the inalienable public domain. Indeed, “occupation thereof in the c oncept of owner, no matter how long, cannot ripen into ownership and be registered as a title.” To

overcome such presumption, incontrovertible evidence must be shown by the applicant. Absent such evidence, the land sought to be registered remains inalienable. Even assuming arguendo that petitioners have been able to prove that the land is alienable, their Petition for confirmation of their imperfect titles and registration thereof under the law will still be denied. The reason is that they have failed to establish possession of the lots in question -openly, continuously, exclusively and adversely -- in the concept of owner for at least 30 years, since June 12, 1945. Petitioners do not claim that they are the original possessors of the lots in question, which had allegedly belonged to Cirilo Menguito before he donated it to his son Pedro. When Pedro died in 1978, these lots allegedly passed down to petitioners. Although petitioners can trace their possession of the land from as far back as 1968 only, they would tack it to that of their predecessors, who had supposedly been in possession thereof even before the Second World War. There is not enough convincing proof, however, to support such claim. 1. Disposition of Agricultural Lands

Public lands suitable for agricultural purposes can be disposed of only as follows, and not otherwise:25 (1) For homestead settlement; (2) (3) (4)

By sale; By lease; By confirmation of imperfect or incomplete titles: (a) By judicial legalization; (b) By administrative legalization (free patent).

Except for lease concessions which do not transfer ownership over land, the result of the foregoing dispositions is the issuance by the government of what is known as a land patent which is generally understood as a grant of land from the government. Applications for these patents, whether by homestead, sale or free patent as outlined in Section 9, are purely administrative in nature and are not proceedings in rem. Thus, a title issued to a grantee, while sufficient to transfer ownership, is not conclusive proof thereof and confers no better right than that conferred by the patent. Instead, under Section 103 of the Property Registration Decree (Presidential Decree No. 1529), such patents are merely evidence of a contract between the Government and the grantee, and are subject to prior vested rights.26 In order to be conclusive or binding as against third parties, therefore, the grantee is required by law to register such patents under the Torrens System.27 It is only upon such registration that actual conveyance of the land is effected.28 25 Section 11, Public Land Act.

26 QUOTE SECTION 103 of PD 1529. 27 Section 103, PD 1529. 28 Section 107, Public Land Act. Page | 18

Furthermore, under the Public Land Act, all patents issued pursuant thereto are subject to the following limitations: (a) Patents or certificates issued shall not include nor convey the title to any gold, silver, copper, iron, or other metals or minerals, or other substances containing minerals, guano, gums, precious stones, coal, or coal oil contained in lands granted thereunder. These shall remain to be property of the State.29 (b) All persons receiving title to lands shall hold such lands subject to the same public servitudes as exist upon lands owned by private persons, including those with reference to the littoral of the sea and the banks of navigable rivers or rivers upon which rafting may be done.30 (c) The said lands shall further be subject to a right-of-way not exceeding sixty (60) meters in width for public highways, railroads, irrigation ditches, aqueducts, telegraph and telephone lines and similar works as the Government or any public or quasi-public service or enterprise, including mining or forest concessionaires, may reasonably require for carrying on their business, with damages for the improvements only.31 (d) The said lands shall also be subject to such rules and regulations as the Government may issued for the beneficial use of water found on these lands,32 including the right to use water flowing through such land for the purposes of power generation.33 Whenever, by priority of possession, rights to the use of water for mining, agricultural, manufacturing, or other purposes have vested and accrued, and the same are recognized and acknowledged by the local customs, or by the laws and decisions of the courts, the possessors and owners of such vested rights shall be maintained and protected, and all patents granted shall be subject to any vested and accrued rights to ditches and reservoirs used in connection with such water rights as may have been acquired. 34 (e) All lands, including homesteads upon which final proof has not been made or approved, shall, even though and while the title remains in the State, be subject to the ordinary taxes, which shall be paid by the grantee or the applicant, beginning with the year next following the one in which the homestead application has been filed, or the concession has been approved, or the contract has been signed, as the case may be, on the basis of the value fixed in such filing, approval or signing of the application, concession or contract.35 Homestead Settlement

By homestead is meant the home, the house and the adjoining land where the head of the family dwells; the home farm; the fixed residence of the head of a family, with the land and buildings surrounding the main house.36 Technically, and under modern homestead laws, it is an artificial estate in land, devised to protect the possession and enjoyment of the owner against the claims of his creditors, by withdrawing the property from execution and forced sale, so long as the land is occupied as a home.37 i.

29 30 31 32 33 34 35 36 37

Qualification and application

Section 110, Public Land Act. Section 111, Public Land Act. Section 112, Public Land Act. Section 113, Public Land Act. Section 114, Public Land Act. Section 113, Public Land Act. Section 115, Public Land Act. Oliver v. Snowden, 18 Fla. 825, 43 Am. Rep. 388. Buckingham v. Buckingham, 8 Mich. 89, 45 N.W. 504. Page | 19

In order to qualify for a homestead settlement, which must not exceed twelve (12) hectares of agricultural land of the public domain following Section 3, Article XII of the 1987 Constitution, 38 the applicant must satisfy the following specific requirements: (a) He must be a citizen of the Philippines over the age of eighteen (18) years, or is the head of a family; 39 (b) He does not own, or has not received by gratuitous allotment from the government, more than twelve (12) hectares of land in the Philippines. 40 Under the Public Land Act, the applicant is only entitled to one homestead entry, and no person to whom a homestead patent has been issued regardless of the area of the original homestead may again acquire a homestead. However, pursuant to Commonealth Act No. 456, any person who had been previously granted a homestead of less than twelve (12) hectares may enter into a subsequent homestead provided that the total area of the homesteads do not exceed twelve (12) hectares.41 (c) He must cultivate and improve at least one-fifth (1/5) of the land continuously within one year and not more than five (5) years from the approval of the homestead application; 42 (d) He must reside in the land for at least one year in the municipality in which the land is located, or in a municipality adjacent to the same; 43 (e) He must commence work on the homestead at least six (6) months from the approval of the homestead application44 and, (f) He must make the application in his own name and that of his family, and not in representation of another person. It has been ruled in the case of  Addu vv. De Yro,45 that a homestead applicant is required by law to occupy and cultivate the land for his own and his family’s benefit, and not for the benefit of someone

else. If he occupies it and cultivates it on behalf of another person and obtains title thereto on the understanding that a portion thereof would be transferred to the latter, such agreement is invalid. It is a ground for cancellation of the entry and bars the issuance of a patent. Upon filing and approval of the homestead application with the Director of Lands, the applicant shall be authorized to take possession of the land upon payment of the corresponding application fee. At the option of the applicant, payment of the fees required for the application may be made to the municipal treasurer who shall, in turn, forward them to the provincial treasurer. In case of delinquency of the applicant, the Director of Lands may, sixty (60) days after such delinquency has occurred, either cancel the application or grant an extension of time not to exceed one hundred and twenty (120) days for the payment of the sum due. 46 Pertinently, it was held in Diaz and Reyes vs.  Macalinao, et al.47  that a homestead entry having been permitted by the Director of Lands, the homestead is segregated from the public domain and the Director Lands is divested of the control and possession thereof except if the application is finally disapproved and the entry annulled or revoked.

38 Section 3, Article XII of the 1987 Constitution provides that citizens of the Philippines may lease not more than five hundred hectares, or acquire more than twelve hectares thereof by purchase, homestead or grant. 39 Section 12 of the Public Land Act. 40 Section 12 of the Public Land Act. 41 Section 19 of the Public Land Act. 42 Section 14 of the Public Land Act. 43 Section 12 of the Public Land Act. 44 Section 13 of the Public Land Act. 45 No. 29449-R, 31 March 1965. 46 Section 15 of the Public Land Act. 47 102 Phil. 999 [1958]. Page | 20

Furthermore, approval of the homestead applicantion grants the applicant a right to occupy the land subject of the homestead. This means that the applicant shall have the right to exercise all rights of legal possession granted to him under the law, including the right to bring an action of forcible entry or unlawful detainer, or any other suitable remedy provided by law. 48

ii.

Entitlement to a homestead patent

Should the applicant comply with the qualifications and obligations prescribed by the Public Land Act, he shall acquire a vested right to the land, and will be entitled to receive a final deed of conveyance called a homestead patent. The execution and delivery of the patent, after the right to a particular parcel of land has become complete, are the mere ministerial acts of the officer charged with that duty. Before final proof is submitted by an applicant of his compliance with the requirements, however, the Public Land Act requires that due notice must first be given to the public of the applicant’s

intention to make such proof, stating therein the name and address of the homesteader, the description of the land, the names of the witnesses by whom it is expected that the necessary facts will be established, and the time and place at which such proof shall be made.49 Proof of possession and cultivation in accordance with the requirements of the Public Land Act is a sine qua non for the grant of a homestead patent, or a vested right in the land. In this regard, it has been held that a mere investigation report submitted by an official of the Bureau of Lands who drew the sketch of the land showing the improvements existing thereon is insufficient to establish the right of an applicant to the land. 50 QUINSAY v. INTERMEDIATE APPELLATE COURT

195 SCRA 268 (1991)

REGALADO, J p:

In a verified complaint dated January 27, 1975, herein private respondents filed Civil Case No. Br. V-216 in the former Court of First Instance of Isabela for annulment of patent, reconveyance and damages against herein petitioners Benito Quinsay, Meliton Caberto and Pastor Olalan, and impleading therein the Director of Lands as a codefendant of the said petitioners. We quote the antecedents which spawned the present controversy, as found by the trial court and thoroughly narrated in its decision of December 13, 1980: "The facts of the case are undisputed. Lot No. 1105, Cad. 211, situated at Divisoria, Santiago, Isabela was the subject of Homestead Application No. V-11083 (E-V-10060) of Beatriz Bayle filed with the Bureau of Lands. Later, she transferred her rights thereto to Sylverio Valdez who, on January 2, 1950, in turn transferred his rights to Fermin Guy Yoche (Exhibits 'F' and 'F-1'). These transfers were approved by the Undersecretary of Agriculture and Natural Resources in his Order of January 31, 1951 (Exhibits 'G' and 'G1'). Obviously, Guy Yoche's homestead application was opposed by defendant Benito Quinsay who was occupying the lot. It was not established how or why it came about but in the course thereof Fermin Guy Yoche and Benito Quinsay entered into an 48 Pitargue v. Sorilla, G.R. No. L-4302, 17 September 1952. 49 Section 17, Public Land Act. 50 Quinsay v. Intermediate Appellate Court, 195 SCRA 268 (1991), citing People v. Sibayan, 116 SCRA 180 (1982) Page | 21

amicable settlement (Exhibits 'A' and 'A-1') concerning the land, agreeing that the northeast corner of the lot, with an area of 1,500 square meters, designated as portion B, shall pertain to Benito Quinsay while the rest of the lot, with an area of 48,765 square meters, designated as portion A, shall pertain to Fermin Guy Yoche. By virtue of such agreement, Benito Quinsay filed on January 20, 1950 a Miscellaneous Sales Application (Exhibits 'L' and 'L-1') over portion B of the lot while Fermin Guy Yoche filed on January 2, 1950 a Homestead Application (Exhibits 'H' and 'H-1') over portion A. The Director of Lands approved the amicable settlement on January 8, 1951 (Exhibits 'B' and 'B-1') and accepted both the Miscellaneous Sales (new) Application of Quinsay and the Homestead Application of Fermin Guy Yoche by decreeing that they shall be 'given further action in accordance with standing rules and regulations on the matter.' On May 22, 1951, plaintiff Urbana Tapiador Vda. de Guy Yoche, in her own behalf and in behalf of the other heirs of Fermin Guy Yoche, filed with the Bureau of Lands their Final Proof (Exhibits 'I' and 'I-1') on the homestead application of her deceased husband. In connection therewith, Asst. Public Land Inspector Hilarion Briones submitted, through the Acting District Land Officer of Isabela, to the Director of Lands his final investigation report recommending the survey and the issuance of a homestead patent to and in favor of the Heirs of Fermin Guy Yoche. This is the last action of the Bureau of Lands taken in so far as the homestead application of Guy Yoche is concerned. xxx "In the meantime, Benito Quinsay and his children, intervenor Julio Quinsay and Florida Quinsay, continued to occupy and possess the whole Lot 1105. Fermin Guy Yoche or any of his heirs never attempted to take possession thereof. Quinsay and his children cultivated and converted into riceland portions thereof in which they planted palay. They also planted vegetables and fruit trees thereon. Then, on March 6, 1972, Benito Quinsay sold to Pastor Olalan 1,250 square meters portion of the land. Likewise, on  January 31, 1974, he again sold another portion, with an area of 3,013 square meters, to Meliton Caberto. There is no showing that Pastor Olalan and Meliton Caberto knew of the homestead application of Fermin Guy Yoche over the land, much more the existence of any amicable settlement over it. What was established is that during all the time that Olalan and Caberto stayed in Divisoria (and they are still there), they only saw Benito Quinsay to be the owner and in possession of the land. After the sales, Olalan and Caberto took actual possession of the portions bought by them, declared their respective portions in their names for taxation purposes and paid real estate taxes thereof. Afterwards, they filed their separate free patent applications (Exhibit 'K' and 'J') over their individual portions with the Bureau of Lands on March 23, 1974 and March 1, 1974, respectively. For purposes of their applications, they submitted the plans (Exhibits 'M' and 'N') of the portions they bought. The District Land Officer of Isabela acted favorably on their applications and in due time issued the corresponding patents thereto. On the strength of these patents, Original Certificate of Title No. P-26759 (Exhibits '9'-Caberto to '9-B'-Caberto) was issued by the Register of Deeds of Isabela to Meliton Caberto on May 24, 1974 while Original Certificate of Title No. P-27214 (Exhibits '11'-Olalan and '11-A'Olalan) was issued to Pastor Olalan on August 16, 1974. It is these two titles which are sought to be annulled by the plaintiffs as Heirs of Fermin Guy Yoche, maintaining that the portions covered thereby was sold by Benito Quinsay to the titled holders in violation of the amicable settlement entered into between him and their predecessor-ininterest. xxx The court a quo, after due ratiocination, and holding that — ". . . In Republic vs. Diamonon, et al., 97 Phil. 843, it was held that 'the right of a homesteader to the land granted to him ripens into a vested right after the filing of the final proof and approval thereof by the Director of Lands (emphasis ours). This doctrine was reiterated in the cases of Lucas vs. Durian, 102 Phil. 1157 and Ingaran vs. Ramelo, 107 Page | 22

Phil. 503, which stated in no uncertain terms that 'an applicant may be said to have acquired a vested right over the homestead only when his application has been perfected by the presentation of the final proof and its approval by the Director of Lands'. In the case at bar, while it is true that the plaintiffs already submitted final proof, the same has not yet been approved by the Director of Lands. As a matter of fact, the Bureau of Lands is still investigating the contending claims of several persons over the land. With more reason then, that the plaintiffs cannot say that they have any vested right over the land. Moreover, the right of the homesteader to a patent does not become absolute until after he has complied with all the requirements of the law ( Vda. de Delizo, et. al. vs. Delizo, et al. , L-32820-21, January 30, 1976, 69 SCRA 216 ; Quiaoit vs. Consolacion, et. al., L-41824, Sept. 30, 1976, 73 SCRA 208). . . ." rendered judgment (1) dismissing the complaint of private respondents, as plaintiffs therein; (2) upholding the validity of Free Patent No. 8223 and Original Certificate of Title No. P-27214 issued in favor of petitioner Pastor Olalan; (3) upholding the validity of Free Patent No. 1110 and Original Certificate of Title No. P-26759 issued in the name of petitioner Meliton Caberto; (4) ordering private respondents not to disturb and molest petitioner Olalan and Caberto in the possession and occupation of the portions owned by the latter; (5) dismissing the counterclaims of petitioner; and (6) dismissing the intervention filed by intervenor Julio Quinsay. Private respondents elevated their case to the then Intermediate Appellate Court seeking reversal of said decision on the contention that the trial court erred in not holding that they and their predecessors had acquired vested rights to the homestead in question. Their plaint found favor in the eyes of respondent court which rendered the following  judgment, and what it considered the rationale therefor, in AC G.R. CV No. 68253: "There appears to be two (2) schools of thought on the matter. The first adopted by the trial court and defendant-appellees, including the Director of Lands, is to the effect that for vested rights to exist the final proof must be not only filed but must be approved by the Director of Lands, citing the cases of Republic vs. Diamonon, 97 Phil. 843, Lucas vs. Durian, 102 Phil. 1157, and Ingaran vs. Ramelo, 107 Phil. 503. The second, subscribed to by plaintiffs-appellants, merely requires a finding by the Bureau of Lands through its proper officials that the homesteader has complied with all the terms and conditions which entitle him to a patent, invoking the landmark doctrine laid down in the case of Balboa vs. Farrales, 51 Phil. 498, reading as follows: 'When a homesteader has complied with all the terms and conditions which entitle him to a patent for a particular tract of land, he acquires a vested interest therein, and is regarded as the equitable owner thereof. Where the right to patent has once become vested in a purchaser of public land, it is equivalent to patent actually issued. The execution and delivery of the patent, after the right to a particular parcel of land has become complete, are the mere ministerial acts of the officer charged with that duty. Even without a patent, a perfected homestead is a property right in the fullest sense, unaffected by the fact that the permanent title to the land is still in the Government. Such land may be conveyed or inherited. No subsequent law can deprive him of that vested right.' xxx

xxx

xxx

"While we do not decide that there is a divergence of opinions between the cases cited by appellees and the case of Balboa vs. Farrales, supra., We vote to follow the Balboa doctrine. Plaintiffs-appellants have already submitted their final proof of possession, residence and cultivation. They have paid the legal fees and charges. Hilarion Briones of the Bureau of Lands in Ilagan, Isabela has recommended survey of the land and the Page | 23

issuance of patent to the heirs of Fermin Guy Yoche, it appearing that all the requirements of law on the matter have been complied with. xxx

xxx

xxx

"WHEREFORE, we hereby reverse and set aside the decision appealed from and render another one annulling as null and void the Free Patents and Original Certificates of Title issued in the names of defendants-appellees Pastor Olalan and Meliton Caberto, ordering defendant-appellee Director of Lands to approve the final proof submitted by plaintiffs-appellants and issue a patent covering Lot 'B', Lot 1105, Cad. Lot No. 211 situated in Divisoria, Santiago, Isabela, with an area of 48,765 square meters, more or less, in the name of the Heirs of Fermin Guy Yoche, and ordering private defendantsappellees, jointly and severally to pay plaintiffs-appellants the sum of P3,000.00 as attorney's fees (Mabutol vs. Mabutol, G.R. No. 60898, September 29, 1983; San Miguel Brewery vs. Magno, 21 SCRA 292; Cabungal vs. Cordova, 11 SCRA 584)." Hence the present appeal by certiorari, petitioners posing for resolution the issues as to whether or not: 1. Private respondents have acquired a vested right over the lots in question; xxx

xxx

xxx

On the first issue, respondent court rejected the theory, submitted by petitioners and the Director of Lands and adopted by the trial court, to the effect that the approval of the final proof of the homestead application of private respondents by the Director of Lands is necessary for vested rights to exist in favor of said respondents over the land in question. Respondent court relied upon some pronouncements in Balboa vs. Farrales where it was allegedly held that where the Bureau of Lands finds through its proper officials that the homesteader has complied with all the terms and conditions which entitle him to a patent, this suffices for vested rights over the lot to exist in favor of the homestead applicant. A perusal of Balboa, however, convinces us that the reliance thereon of respondent court is misplaced as, in that case, there was approval by the Director of Lands of the final proof of the applicant. Furthermore, what was involved there was a homestead application filed and approved under the aegis of Act No. 926, before its repeal by Act No. 2874. As explained therein, in 1913, Balboa filed a homestead application under the provisions of Act No. 926, and "in 1918, Balboa submitted final proof, showing his residence upon, and cultivation of said land, as well as his compliance with all of the other requirements of Section 3 of said Act No. 926, which final proof was approved by the Director of Lands on February 15, 1918 (Exhibit 3). On July 1, 1919, said Act No. 926 was repealed by Act No. 2874." Thus the Court took pains to specifically point out in said case that — "Section 3 of Act No. 926 provides, inter alia, that upon the final proof by the applicant and the approval thereof by the Director of Lands, 'he (the applicant) shall be entitled to patent' or certificate of title. Therefore, on February 15, 1918, after Buenaventura Balboa had submitted his final proof and after the same had been approved by the Government, and while Act No. 926 was still in force, he became the owner of the land and entitled to a patent. At least on that date his right to the land, as owner, had ripened into a vested right . . . xxx

xxx

xxx Page | 24

"The fact that the homestead patent or certificate of title was issued on September 10, 1920, after the repeal of Act No. 926, and under the provisions of Section 116 of the repealing Act No. 2874 cannot prejudice the vested right acquired by Buenaventura under the provisions of the former Act. (Emphasis supplied.). This had to be so clarified since in said case Balboa sold the land on August 11, 1924, which was within the 5-year prohibition therefor in Section 116 of Act No. 2874, and said sale was sought to be nullified. Hence, the need for the Court to expound on the fact that, under the law and the circumstances obtaining in said case, Balboa had acquired vested rights to the land even before the approval of his final proof. That was the rule adopted under the law then in force, and the reason for the judicial discourse on vested rights in the decision in said case supposedly because the provisions of Act No. 926 did not negate the same. Obviously, however, said pronouncements, albeit in effect obiter dicta, were further truncated and applied out of the context by respondent court. We need not repeat the contrary and present rule under the subsequent law and  jurisprudence on the matter, as catalogued by the trial court and set out earlier in this opinion. Now, in the case at bar, the homestead application and the final proof submitted by the heirs of Fermin Guy Yoche on May 22, 1951 remain unapproved up to the present. From what has been said, therefore, vested rights over the land cannot be validly claimed by private respondents since the approval by the Director of Lands of their final proof for a homestead patent is now a condition sine qua non for the existence of such vested right. Thus, in the aforecited case of Republic vs. Diamonon, et al., this Court, noting that in espousing the contrary view Diamonon invoked the provisions of Act No. 926 and sought refuge in Balboa, rejected that submission since "(t)he fallacy of appellant's argument lies in the failure to consider the facts of his citation, wherein final proof was approved by the Government while Act No. 926 was effective. In the case now before us, it was only on November 9, 1933, that the Director of Lands approved the appellant's final proof or about fourteen years after the enactment of Act No. 2874 on November 29, 1919." xxx

xxx

xxx

WHEREFORE, the judgment of respondent Intermediate Appellate Court is hereby REVERSED and SET ASIDE and the decision of the trial court is REINSTATED. SO ORDERED. Melencio-Herrera, Paras, Padilla and Sarmiento, JJ., concur.

BALBOA vs.FARRALES, 51, Phil. 498 (1928)  JOHNSON, J.:

The material facts in this case, as disclosed by the record, may be briefly stated as follows. (1) Sometime in the year 1913, the plaintiff Buenaventura Balboa filled with the Bureau of Lands an application for homestead, No. 10619, under the provisions of Act No. 926, covering a tract of land situated in the barrio of Culis, municipality of Hermosa, Province of Bataan, containing 14 hectares, 49 ares and 77 centares. (2) Five years thereafter, or in 1918, Balboa submitted proof, showing his residence upon, and cultivation of said land, as well as his compliance with all of the other requirements of section 3 of said Act No. 926, which final proof was approved by the Director of Lands on February 15, 1918 (Exhibit 3). On July 1, 1919, said Act No. 926 was repealed by Act No. 2874. Page | 25

(3) On September 10, 1920, or over a year after Act No. 2874 had gone into effect, the homestead patent for said land, otherwise known as certificate of title No. 91 (Exhibit A) was issued n favor of Buenventura Balboa by the Governor-General of the Philippine Islands. (4) On August 11, 1924, said Buenaventura Balboa, for and in consideration of the sum of P950, sold said land to the defendant Cecilio L. Farrales (Exhibit 2); and on October 16, 1924, the latter secured in his name transfer certificate of title No. 650 of said land (Exhibit B). On March 6, 1926, the plaintiff commenced the present action for the purpose of having said sale declared null and void on the ground of lack of consent on his part and fraud on the part of the defendant, and on the further ground that said sale was contrary to, and in violation of the provisions of section 116 of Act No. 2874. After a careful consideration of the evidence adduced during the trial of the cause the Honorable Leopoldo Rovira, judge, arrived at the conclusion that the deed of sale in question (Exhibit 2) had been duly executed by the plaintiff. He held, however, that said deed was null and void, in view of the fact that it was executed before the lapse of five years from the date of the issuance of the certificate of title in favor of Buenventura Balboa, in violation of the prohibition contained in section 116 of Act No. 2874. The pertinent parts of the decision read as follows: Como cuestion basica, se discute en el presente asunto la validez del documento Exhibit 2, o sea el traspaso hecho por el demandante al demandado referente al terreno en cuestion. El demandante sostiene que, bajo el articulo 116 de la Ley 2874, el traspaso el nulo por cuanto tuvo lugar el 11 de agosto de 1924, esto es sin haber transcurrido todavia los cinco anos siguientas a la fecha en que fue expedidol el certificado de titulo No. 91 que lo fue el 10 de septiembre de 1920; el demandado, por el contrario, sostiene, como punto de discusion legal, que el documento de traspaso exhibit 2 no cae bajo las disposiciones de la Ley No. 2874, sino dentro de las disposiciones de la Ley No. 926 y que bajo esta Ley no existia tal limitacion de venta dentro de los cinco años siguientes a la fecha de la expedicion del titulo dehomestead, y que habiendo sido la solicitud de homestead aprobada 15 de febrero de 1918, aun contado los cinco anos siguientes, resultaria que desde el 15 de febrero de 1918 hasta el 11 de agosto de 1924 han transcurrido mas de cinco años. xxx

xxx

xxx

De lo expuesto, el Juzgado Ilega a la conclusion de que el Exhibit 2 es nulo e ineficaz, por cuanto que la venta fue otorgada fuera de lo prescrito en el articulo 116 de la Ley No. 2874, que procede declarar nulo dicho documento Exhibit 21, y, consiguintemente, el certificado de transferencia de titulo 650. In accordance with the foregoing conclusion the trial judge rendered a judgment in favor of the plaintiff and against the defendant, ordering the latter to return to the plaintiff the land in question, and the plaintiff to return to the defendant the price received for said land, aggregating P652.69, with interest at the rate of 12 per cent. From the judgment both parties appealed. The principal question raised in this appeal is whether the validity of the sale of the land in question should be determined under the provisions of Act No. 926 or under those of Act No. 2874. In other words, which of the two Acts — 926 and 2874 — shall be applied in determining whether the sale in question is valid or not? The land in question was acquired by Buenventura Balboa as homestead under the provisions and pursuant to the requirements of Act No. 926. He filed his application and complied with all of the requisites to the acquisition of said homestead, in conformity with the provisions of said Act No. 926. In 1918 and prior to the repeal of said Act he submitted his final proof, showing his residence upon, and cultivation of the land, as well as his compliance with all of the other requirements of the law, and said final proof was approved by the Director of Lands on February 15, 1918. In other words, Buenaventura Balboa, had shown, to the satisfaction of the Government, that he had performed all of the acts required of an applicant for homestead, and, under the provisions of section 3 of Act no. 926, he became entitled to a homestead patent or certificate of title to the land covered by his application. Section 3 of Act No. 926 provides, inter alia, that upon the filing of final proof by the applicant and the approval thereof by the Director of Lands, "he (the applicant) shall be entitled to a patent" Page | 26

or certificate of title. Therefore, on February 15, 1918, after Buenaventura Balboa had submitted his final proof and after the same had been approved by the Government, and while Act No. 926 was still in force, he became the owner of the land and "entitled to a patent." At least on that date his right to the land, as owner, ripened into a vested right. It was no longer expectant as depending on the continuance of existing circumstances, or contingent as depending on some events or the performance of some conditions. Rights are vested when the right to enjoyment, present or prospective, has become the property of some particular person or persons as a present interest. (12 C. J., sec. 485, p. 955.) Vested right "is some right or interest in property which has become fixed and established and is no longer open to doubt or controversy." (Downs vs. Blount, 170 Fed. Rep., 15, 20.) The fact the homestead patent or certificate of title No. 91 was issued on September 10, 1920, after the repeal of Act No. 926, and under the provisions of section 116 of the repealing Act No. 2874, cannot prejudice the vested right acquired by Buenventura Balboa under the provisions of the former Act. The issuance of the certificate of title was a mere ministerial act, and the certificate, an outward symbol of his vested right to the land, of which he was virtually recognized as owner by the Government on February 15, 1918. In the case of United States vs. Freyberg (32 Fed. Rep., 195), where the right of a homesteader was involved, it was held that where the right to a patent for land has become vested in a purchaser the Government holds the legal title in trust for the purchaser until the patent is issued. Again in the case of Stark vs. Starr (6 Wallace [U. S.], 402), the Supreme Court of the United States held that where the right to a patent is once vested, it is treated by the Government, when dealing with public lands, as equivalent to a patent issued. A party who was has complied with all the terms and conditions which entitle him to a patent for a particular tract of public land acquires a vested interest therein, and is to be regarded as the equitable owner thereof. (Wirth vs. Branson, 98 U. S. 118.) Where the right to a patent has once become vested in a purchaser of public lands, it is equivalent so far as the Government is concerned, to a patent actually issued. The execution and delivery of the patent after the right to it has become complete are the mere ministerial acts of the officers charged with that duty. (Simmons vs. Wagner 101 U. S., 260.) The moment the plaintiff had received a certificate from the Government and had done all that was necessary under the law to secure his patent, his right had become vested before the patent was issued. His right had already vested prior to the issuance of the patent, and his rights to the land cannot be affected by a subsequent law or by a subsequent grant by the Government to any other person. (Herron vs. Dater, 120 U. S., 464.) The delay in the issuance of the patent cannot affect the vested right of the homesteader. (Murphy vs. Packer, 152 U. S., 398; Belk vs. Meagher, 104 U. S., 279; Sullivan vs. Iron Silver Mining Co., 143 U. S., 431; McDaniel vs. Apacible and Cuisia, 42 Phil., 749.) A perfected valid appropriation of public land operates as a withdraw of the tract from the body of the public domain and, so long as such appropriation remains valid and subsisting the land covered thereby is deemed private property. A perfected homestead, under the law, is property in the highest sense, which may be sold and conveyed and will pass by descent. It has the effect of a grant of the right to present and exclusive possession of said land. A valid and subsisting perfected homestead, made and kept up in accordance with the provisions of the statute, has the effect of a grant of the present and exclusive possession of the land. Even without a patent, a perfected homestead is a property right in the fullest sense, unaffected by the fact that the paramount title to the land is in the Government. Such land may be conveyed or inherited. In the United States and in each and every State of the Union vested rights are safeguarded by the 4th Amendment to the Federal Constitution, which provides that no State "shall deprive any person of life, liberty or property without due process of law." The state has no power to divest or to impair vested rights, whether such an attempt to do so be made by legislative enactment, by municipal ordinance, or by a change in the constitution of the estate. This result follows from prohibitions contained in the constitution or particularly all the states. Before the adoption of the fourteenth amendment there was no prohibition in the Constitution of the United States which would prevent the states from passing laws divesting Page | 27

vested rights, unless these laws also impaired the obligation of contact, or were ex post facto laws; but vested property rights are now protected against state action by the provision of the fourteenth amendment that no state "shall deprive any person of life, liberty or property without due process of law ." (12 C. J., sec. 486, pp. 956, 957.) Section 3, paragragh 1, of the Jones Law provides: "That no law shall be enacted in said Islands which shall deprive any person of life, liberty, or property without due process of law, etc." Thus, in this jurisdiction, vested rights are also protected from impairment by express constitutional provision. Therefore, the right vested in Buenaventura Balboa by Act No. 926 cannot be divested, impaired or restricted by section 116 of Act No. 2874. Said right should be governed entirely and exclusively by the provisions of Act No. 926, which it was acquired. Now, the vested right of Buenaventura Balboa to his homestead land necessarily carries with it the right to alienate and dispose of the same. The only prohibition contained in Act No. 926 against alienation of homestead acquired under said law, appears in section 4 thereof, which reads as follows: "No lands acquired under the provisions of this chapter shall in any event become liable to the satisfaction of any debt contracted prior to the issuance of a patent therefor." It follows, therefore that the sale of the land in question by the plaintiff Buenventura Balboa to the defendant Cecilio L. Farrales does not infringe said prohibition, and consequently said sale is valid and binding, and should be given full force and effect. Section 116 of Act No. 2874, which prohibits the sale of homestead land during the period of five years subsequent to the issuance of the patent or certificate of title upon which rests the decision of the court a quo, cannot be invoked to annul the sale in question. Said prohibition, if applied in the present case, would impair and diminish the vested rights acquired under Act No. 926, contrary to the uniform doctrine followed in the United States, and in violation of the express provisions of section 3 of the Jones Law. The right, title and interest of the appellant having become vested under the provisions of Act No. 926, his rights cannot be affected by any law passed subsequent thereto. The provisions of Act No. 2874 cannot be invoked for the purpose of defeating the vested right acquired by the appellant before its adoption. For all of the foregoing reasons, the judgment appealed from should be and is hereby reversed, and it is hereby ordered and decreed that the defendant be absolved from all liability under the complaint, with costs against the plaintiff-appellant. So ordered.  Malcolm, Villamor, Ostrand and Villa- Real, JJ., concur.

iii.

Conditions and limitations on the rights of the grantee of a homestead patent

In addition to the general conditions applicable to grantees of patents provided in Chapter XIII of the Public Land Act and enumerated above, holders of homestead patents are similarly subject to the following special conditions and limitations: (a) Land covered by a homestead patent cannot be alienated or encumbered from the date of the approval of the application and for a term of five (5) years from and after the date of issuance of the patent, except if such alienation or encumbrance is made in favor of the government or any of its branches, units or institutions, or to legally constituted banking corporations.51 (b) Land covered by a homestead patent cannot be held liable in satisfaction of any debt contracted prior to the said period. However, improvements or crops on the land may be mortgaged or pledged to qualified persons, 52 who, in accordance with Section 122,53 must be Filipino citizens or corporations. 51 Section 118 of the Public Land Act. 52 Section 118 of the Public Land Act. 53 Section 122 of the Public Land Act provides: Page | 28

(c) Land covered by a homestead patent shall not be alienated, transferred or conveyed after five (5) years and before twenty-five (25) years after issuance of the patent unless approved by the Secretary of Environment and Natural Resources, which approval cannot be denied except on Constitutional grounds.54 (e) Land covered by a homestead patent, or any permanent improvement thereon, cannot be alienated or transferred to a corporation, association or partnership without the consent of the grantee and approval of the Secretary of Environment and Natural Resources, and solely for commercial, industrial, educational, religious or charitable purposes or for a right of way.55 (d) Every conveyance of land acquired under a homestead patent shall be subject to repurchase by the applicant, his widow or legal heirs, within a period of five (5) years from the date of the conveyance. 56 In Evangelista v. Montano, et al. , the requirement of obtaining approval from the Secretary of Environment and Natural Resources for sales of land covered by homestead patent after five years from the issuance of the title was held to be merely directory, and may be complied with at any time in the future. Thus, in one case where a homestead was sold after the expiration of five (5) years and transfer took place before the expiration of twenty-five (25) years, from the issuance of the patent, the failure to secure the requisite approval from the Secretary of Environment and Natural Resources did not render the sale null and void. 57 On the other hand, unless made in favor of the government, or any of its branches, units or institutions, or to legally constituted banking corporations, the sale of land within the 5-years from the issuance of the homestead patent renders such sale null and void. The reason for this prohibition is to “preserve and keep in the family of the homesteader that portion of public l and which the State has gratuitously given to them.” Thus, in Egao v. Court of Appeals, the Supreme Court ruled that the sale of a homestead patent within the 5-year prohibited period if null and void. In this case, the Supreme Court allowed the heirs to recover the said parcel of land, notwithstanding the possible application of the  pari delicto doctrine. The Court reasoned that the application of the  pari delicto doctrine does not apply in cases where the sale is void ab initio under the Public Land Act, as when its enforcement or application runs counter to the public policy of preserving the grantee’s right to the land under the homestead patent. EGAO , ET AL. VS. COURT OF APPEALS , ET AL., 174 SCRA 484 [1989.]

PADILLA, J p: This is a land dispute which culminated in the filing by private respondents Severo Dignos and Severo Bontilao of a verified complaint for Quieting of Title and/or Recovery of Possession and Ownership before the RTC of Manolo Fortich, Bukidnon, * against petitioners Apolonio and Beatriz Egao.

“No land ori ginally acquired in any manner under the provisions of this Act, nor any permanent improvement on

such land, shall encumbered, alienated, or transferred, except to persons, corporations, associations, or partnerships who may acquire lands of the public domain under this Act or to corporations organized in the Philippines authorized therefor by their charters. Except in cases of hereditary succession, no land or any portion thereof originally acquired under the free patent, homestead, or individual sale provisions of this Act, or any permanent improvement on such land, shall be transferred or assigned to any individual, nor shall such land or any permanent improvement thereon be leased to such individual, when the area of said land, added to that of his own, shall exceed one hundred and forty-four hectares. Any transfer, assignment, or lease made in violation hereof, shall be null and void.”

54 55 56 57

Section 118 of the Public Land Act. Section 121 of the Public Land Act. Section 119 of the Public Land Act. Flores, et al. v. Plasina, et al., L-5727, Feb. 12, 1954. Page | 29

Private respondents' complaint alleged that they are the legitimate owners and possessors of two (2) parcels of land situated at Lonocan, Manolo Fortich, Bukidnon, per deed of absolute sale dated 21 December 1979 which, among others, recited thus: xxx

xxx

Allegedly, upon purchase of Lot No. 662 from Roberto Marfori, improvements were introduced and taxes paid by private respondents. Sometime in June 1983, herein petitioners allegedly occupied illegally portions of the land. Petitioners' answer to the complaint asserted that Apolonio Egao is the registered owner of the parcel of land known as Lot No. 662, Pls 854 with an area of 3,451 sq. meters evidenced by OCT No. P-3559 issued by the Register of Deeds of Bukidnon pursuant to Free Patent No. 298112 dated 12 August 1965; that he (Apolonio Egao) and his family have been in actual, physical, adverse, open and continuous possession thereof even before the issuance to him of the free patent; that the land has never been sold by reason of the prohibition against alienation under Commonwealth Act No. 141 (Public Land Law); and that the instant case was the fourth in a series filed against the Egaos and is part of respondents' scheme to grab said parcel of land from the petitioners.  Judge Felicidario M. Batoy ruled in favor of the Egaos, herein petitioners (defendants in the court a quo), ordering respondent Severo Bontilao (plaintiff in the court a quo) to immediately deliver to the Egaos the owner's duplicate copy of Original Certificate of Title No. P-3559. Said trial judge held: "In the instant case, granting arguendo, that defendants executed the 2 documents in favor of Marfori (Exhs. A & B) after the filing of the application for free patent but before the issuance of the latter, without the approval of the Director of Lands, upon issuance of Free Patent No. 298112. On August 12, 1965, the said deeds of sale (Exhs. A & B) were ipso facto cancelled or superseded by said free patent. Moreover, it appears from the evidence that defendants never vacated or abandoned their possession of Lot No. 662 as they have continuously lived on said lot since 1950, a fact admitted by the plaintiffs themselves. And as long as Original Certificate of Title No. P-3559 remains in the name of defendant Apolonio Egao, married to Beatriz Menoza Egao, this is the ultimate and best evidence of title granted by the government which must be honored and respected by the courts. In a nutshell, the plaintiffs miserably failed to present or show any title to Lot No. 662, PLS-854 which should be quieted or freed from any cloud of doubt as prayed for in their complaint and they further failed to show that they are entitled to the ownership and possession to Lot No. 662, PLS-854." Private respondents went to the Court of Appeals in CA-G.R. No. 09539. Setting aside the RTC decision, the appellate court ** held, in part, thus — "That the land is titled in the name of defendant Apolonio Egao is not in question. The main point in issue is whether defendants could validly sell the land to Marfori who in turn transferred ownership thereof to the plaintiffs." Marfori and Egao were both held by the Court of Appeals in pari delicto for violating the five (5) year restriction under Sec. 118, Commonwealth Act No. 141 as amended by Act No. 496 against encumbrance or alienation of lands acquired under a free patent or homestead; hence, they cannot, according to the appellate court, seek affirmative relief, but respondents on the other hand were declared innocent purchasers for value who Page | 30

obtained the owner's duplicate copy of the OCT (still in the name of the Egaos) from Marfori who transferred to them (respondents) physical possession of the property. Finally, the Court of Appeals held: xxx

xxx

xxx

Validity of the Deeds of Sale executed between Marfori (as purchaser) and the petitioners (as sellers) is the main issue to be resolved, in determining respondents' right over the disputed land, the respondents being the transferees of Marfori. It is undisputed that Free Patent No. 298112 was issued to petitioner Apolonio Egao over Lot No. 662 on 12 August, 1965. Sec. 118 of Commonwealth Act No. 141, as amended, prohibits the alienation or encumbrance, within a period of five (5) years from the date of issuance of the patent, of lands acquired under free patent or homestead. Assuming, arguendo, the authenticity of the Deeds of Sale executed by the Egaos in favor of Marfori over portions of Lot No. 662 (the land in question), dated 7 May 1964, 14 January and 6 October 1965, it clearly appears that all deeds were executed within the prohibited period of five (5) years. As correctly found by the appellate court — "Section 124 of the Public Land Act provided [sic] that any acquisition, conveyance, alienation, transfer or other contract made or executed in violation of any of the provisions of Sections 118, 121, 120 122 and 123 of this Act shall be unlawful, null and void from its execution and shall produce the effect of annulling and cancelling the grant, title, patent or permit originally issued, recognized or actually or prescriptively, and cause the reversion of the property and its improvements to the state." Petitioners deny the authenticity and due execution of the notarized deeds of sale in favor of Marfori, asserting continued ownership over the land by virtue of a Torrens Certificate of Title issued in their name. While the Court is not satisfied with respondents' explanation of their failure to present the notaries public (who were residents of a neighboring province) to affirm their participation in the preparation of the Deeds, the Court also finds as insufficient the mere denials by petitioners as to due execution and authenticity of said Deeds of Sale. A notarial document is evidence of the facts in clear unequivocal manner therein expressed. It has in its favor the presumption of regularity. To contradict all these, there must be evidence that is clear, convincing and more than merely preponderant. The question of authenticity being one of fact, the Court will not disturb the conclusions of the Court of Appeals on the matter. Original Certificate of Title No. P-3559 over the land in dispute was issued on 1 March 1966, a few months after the execution by the Egaos of the last Deed of Sale in favor of Marfori. The OCT is registered in the name of the Egaos, herein petitioners. A Torrens title, once registered, cannot be defeated, even by adverse open and notorious possession. A registered title under the Torrens system cannot be defeated by prescription. The title, once registered, is notice to the world. All persons must take notice. No one can plead ignorance of the registration. Contrary to the appellate court's conclusion, respondents are not innocent purchasers for value. An "innocent purchaser for value" is deemed, under the Torrens system, to include an innocent lessee, mortgagee or other encumbrancer for value. Where a purchaser neglects to make the necessary inquiries and closes his eyes to facts which should put a reasonable man on his guard as to the possibility of the existence of a defect in his vendor's title, and relying on the belief that there was no defect in the title of the vendor, purchases the property without making any further investigation, he cannot claim that he is a purchaser in good faith for value. Page | 31

Furthermore, a private individual may not bring an action for reversion or any action which would have the effect of cancelling a free patent and the corresponding certificate of title issued on the basis thereof, with the result that the land covered thereby will again form part of the public domain, as only the Solicitor General or the officer acting in his stead may do so. The rule of  pari delicto non oritur actio (where two persons are equally at fault neither party may be entitled to relief under the law), admits of exceptions and does not apply to an inexistent contract, such as, a sale void ab initio under the Public Land Act, when its enforcement or application runs counter to the public policy of preserving the grantee's right to the land under the homestead law. Sec. 51, par. 2 of the Property Registration Decree (PD 1529), formerly Sec. 50 of the Land Registration Act (Act No. 496) expressly provides that the registration of the Deed is the operative act that binds or affects the land insofar as third persons are concerned. The law requires a higher degree of prudence from one who buys from a person who is not the registered owner, when the land object of the transaction is registered land. While one who buys from the registered owner need not look behind the certificate of title, one who buys from another who is not the registered owner is expected to examine not only the certificate of title but all factual circumstances necessary fo r him to determine if there are any flaws in the title of the transferor, or in his capacity to transfer the land. Failing to exercise caution of any kind whatsoever is tantamount to bad faith. Deeds of sale of patented lands, perfected within the prohibited five (5) year period are null and void (Sec. 124, Public Land Act). No title passed from the Egaos to Marfori which could be validly transferred to herein respondents Bontilao and Dignos. Nemo dat quod non habet (nobody can dispose of that which does not belong to him). While the government has not taken steps to assert its title, by reversion, to a homestead sold in violation of the Public Land Act, the vendor or his heirs is better entitled to the possession of the land, the vendee being in no better situation than any intruder. Accordingly, respondents who are not innocent purchasers for value have no standing to question petitioners' right to the land and to file an action for quieting of title. WHEREFORE, the appealed decision of the Court of Appeals in CA G.R. CV No. 09539 is REVERSED and SET ASIDE. Meanwhile, petitioners as registered owners are entitled to remain in physical possession of the disputed property. Respondents are ordered to deliver the owner's duplicate copy of the OCT (No. P-3559) to petitioners, without prejudice to an action for reversion of the land, which may be instituted by the Solicitor General for the State. Let a copy of this decision be furnished the Solicitor General. SO ORDERED. Melencio-Herrera, Sarmiento and Regalado, JJ., concur. Paras, J., took no part. This rule applies even if approval from the Secretary of Enviroment and Natural Resources is subsequently obtained. Such approval did not have the effect of curing the violation of a mandatory provision of the law.58

58 De los Santos vs. Roman Catholic Church of Midsayap, et al., 90 Phil. 405 [1954] Page | 32

DE LOS SANTOS VS. ROMAN CATHOLIC CHURCH OF MIDSAYAP , ET AL.

94 Phil. 405 [1954]

BAUTISTA ANGELO, J p: On December 9, 1938, a homestead patent covering a tract of land situated in the municipality of Midsayap, Province of Cotabato, was granted to Julio Sarabillo and on March 17, 1939, Original Certificate of Title No. RP-269 (1674) was issued in his favor. On December 31, 1940, Julio Sarabillo sold two hectares of said land to the Roman Catholic Church of Midsayap for the sum of P800 to be dedicated to educational and charitable purposes. It was expressly agreed upon that the sale was subject to the approval of the Secretary of Agriculture and Natural Resources. In December, 1947, a request for said approval was submitted in behalf of the Roman Catholic Church by Rev. Fr. Gerard Mongeau stating therein that the land would be used solely for educational and charitable purposes. The sale was approved on March 26, 1949, and on March 29, 1950, the dead of sale was registered in the Office of the Register of Deeds for the Province of Cotabato. No new title was issued in favor of the Roman Catholic Church although the deed was annotated on the back of the title issued to the homesteader. In the meantime, Julio Sarabillo died and intestate proceedings were instituted for the settlement of his estate and Catalina de los Santos was appointed administratrix of the estate. And having found in the course of her administration that the sale of the land to the Roman Catholic Church was made in violation of section 118 of Commonwealth Act No. 141, the administratrix instituted the present action in the Court of First Instance of Cotabato praying that the sale be declared null and void and of no legal effect. In their answer defendants claim that the sale is legal and valid it having been executed for educational and charitable purposes and approved by the Secretary of Agriculture and Natural Resources. They further claim that, even if it be declared null and void, its immediate effect would be not the return of the land to appellee but the reversion of the property to the State as ordained by law. Defendants also set up as a defense the doctrine of pari delicto. xxx

xxx

xxx

After the parties had submitted the case on the pleadings, in addition to the report of the clerk of court as to the value of the improvements existing on the land, the court rendered decision declaring the sale null and void and ordering the plaintiff to reimburse to the defendants the sum of P800 which was paid as purchase price, plus the additional sum of P601 as value of the improvements, both sums to bear interest at 6 per cent per annum from the date of the complaint, and ordering defendants to vacate the land in question. Dissatisfied with this decision, the case was taken to the Court of Appeals but it was later certified to this Court on the ground that the appeal merely involves questions of law. It appears that the patent covering the tract of land which includes the portion now disputed in this appeal was issued to the late Julio Sarabillo on December 9, 1938, and the sale of the portion of two hectares to the Roman Catholic Church took place on December 31, 1940. This shows that the sale was made before the expiration of the period of five years from the date of the issuance of the patent and as such is null and Page | 33

void it being in contravention of section 118 of Commonwealth Act No. 141. The fact that it was expressly stipulated in the deed of sale that it was subject to the approval of the Secretary of Agriculture and Natural Resources and the approval was sought and obtained on March 26, 1949, or more than ten years after the date of the issuance of the patent, or the fact that the deed of sale was registered in the Office of the Register of Deeds only on March 29, 1950, and was annotated on the back of the title on that date, cannot have the effect of validating the sale for the reason that the approval of the Secretary of Agriculture and Natural Resources does not have any valid curative effect. That approval is merely a formality which the law requires if the sale is effected after the term of five years but before the expiration of a period of 25 years for the purpose of testing the validity of the sale on constitutional grounds. But, as was ruled by this Court, the absence of such formality will not render the transaction null and void ( Evangelista vs. Montaño, G. R. No. L-5567). What is important is the period within which the sale is executed. The provision of the law which prohibits the sale or encumbrance of the homestead within five years after the grant of the patent is mandatory. This cannot be obviated even if official approval is granted beyond the expiration of that period, because the purpose of the law is to promote a definite public policy, which is "to preserve and keep in the family of the homesteader that portion of public land which the State has gratuitously given to him." [ Pascua vs. Talens, 2 45 Off. Gaz., No. 9, (Supplement) 413.] The claim that the sale can be validated because it was made with the avowed aim that the property would be dedicated solely to educational and charitable purposes is likewise unmeritorious even considering the law invoked by counsel for appellants in favor of its validity. It is true that under section 121, Commonwealth Act No. 141, a corporation, association, or partnership may acquire any land granted as homestead if the sale is done with the consent of the grantee and the approval of the Secretary of Agriculture and Natural Resources and is solely for commercial, industrial, educational, religious, or charitable purposes, or for a right of way, and apparently there is no limitation therein as to the time within which such acquisition may be made. But this provision should be interpreted as a mere authority granted to a corporation, association or partnership to acquire a portion of the public land and not as an unbridled license to acquire without restriction for such would be giving an advantage to an entity over an individual which finds no legal justification. It is our opinion that the authority granted by section 121 should be interpreted as subject to the condition prescribed in section 118, namely, that the acquisition should be after the period of five years from the date of the issuance of the patent. xxx

xxx

xxx

Wherefore, the decision appealed from is affirmed, without pronouncement as to costs. Paras, C. J., Pablo, Bengzon, Montemayor, Jugo and Labrador, JJ., concur. Padilla, J., concurs in the result. Furthermore, the 5-year prohibition on the sale of land subject of a homestead patent has been extended to improvements on the land as well. Thus, in Basaltos v. Esteban, Jr., et al. ,59 the Supreme Court said that improvements on lots applied for as homestead cannot be transferred, on pain of nullity, without the approval of the Secretary of Environment and Natural Resources (formerly, Minister of Natural Resources), and the duty to secure such approval devolves upon the vendor because it is he who should give the vendee a clear title to the property he is conveying.

59 No. L-09121, 11 April 1956. Page | 34

In the same manner, as the Public Land Act grants homestead patent holders a right of repurchase by him, his widow or legal heirs, within a period of five (5) years from the date of the conveyance. This right cannot be waived, and any waiver so made shall be considered void as against public policy.60 Furthermore, this 5-year right of redemption is counted separately from redemption rights provided for in other laws, such as Act No. 313561 which grants a mortgagor, his successors, or any judicial creditor a period of one (1) year from an extrajudicial foreclosure sale to redeem a subject property.62 The five-year period fixed in Section 119 begins to run from the expiration of the one-year redemption period. Thus, in Rural Bank of Davao City, Inc. v. Court of Appeals ,63 the Supreme Court said: “Thus, the rules on redemption in the case of an extrajudicial foreclosure of land

acquired under free patent or homestead statutes may be summarized as follows: . . . If the land is mortgaged to parties other than rural banks, the mortgagor may redeem the property within one (1) year from the registration of the certificate of sale pursuant to Act No. 3135. If he fails to do so, he or his heirs may repurchase the property within five (5) years from the expiration of the redemption period also pursuant to Section 119 of the Public Land Act.”

In various cases,64 the Supreme Court held that this right of repurchase is available at any time land subject of a free patent is conveyed to third parties. In Santana, et al., vs. Marinas,65 however, the Supreme Court denied the right of repurchase under Section 119 because its exercise, under the circumstances set forth in the case, was contrary to the philosophy behind Section 119.

SANTANA , ET AL. VS. MARINAS , 94 SCRA 853 [1979]

SANTOS, J p: This is a petition for review by certiorari under Rule 45 of the Revised Rules of Court of the decision dated September 5, 1972 of the Special Sixth Division of the Court of Appeals — composed of Justice E. Soriano, M. Barcelona and L.B. Reyes — in CA-G.R. 37292-R, entitled "Sotero Mariñas vs. Francisco Santana and Jose H. Panganiban", Soriano, J., ponent, which (1) reversed the decision of the Court of First Instance of Rizal dismissing the complaint of Sotero Mariñas — plaintiff below and private respondent herein - for recovery of the property in litigation under Section 119 of Com. Act No. 141, otherwise known as Public Land Law and (2) ordered Francisco Santana and Jose H. Panganiban — defendants below and now herein petitioners — to reconvey the aforesaid property to respondent Mariñas upon payment by him of the repurchase price in the amount of P4,128.60, without special pronouncement as to costs. xxx

xxx

xxx

60 Isaac vs. San Juan Leon, 61 Entitiled, “An Act to Regulate the Sale of Property under Special Powers Inserted in or annexed to Real -Estate Mortgages”

62 Pertinently, Section 6 provides: SECTION 6. In all cases in which an extrajudicial sale is made under the special power hereinbefore referred to, the debtor, his successors in interest or any judicial creditor or judgment creditor of said debtor, or any person having a lien on the property subsequent to the mortgage or deed of trust under which the property is sold, may redeem the same at any time within the term of one year from and after the date of the sale; and such redemption shall be governed by the provisions of sections four hundred and sixty-four to four hundred and sixty-six, inclusive, of the Code of Civil Procedure, in so far as these are not inconsistent with the provisions of this Act. 63 217 SCRA 554 [1993]. 64 Isaac, et al. v. Tan Chuan Leong, et al., 89 Phil. 24 [1951]; Francisco v. Certeza, Sr., 3 SCRA 565 [1961] 65 94 SCRA 853 [1979] Page | 35

The procedural, as well as the factual, antecedents that spawned the present case are not in dispute. On April 21, 1960, private respondent Sotero Mariñas — as plaintiff — filed in the Court of First Instance of Rizal a complaint to recover a real property alleging, insofar as material to this petition, (1) that he acquired, on May 22, 1929, under free patent and covered by Original Certificate of Title (OCT) No. 217, Rizal Registry, a parcel of land containing an area of four hectares, twelve ares and eightysix centares (41,286 sq. m.); that on January 16, 1956, he sold the above parcel of land to petitioner Francisco Santana — one of the defendants below — for a sum of P4,128.60; that the other petitioner Jose H. Panganiban — also a co-defendant below - was included in the complaint because he is a subsequent lienholder and/or encumbrancer, the property having been sold to him by Santana on March 25, 1956 for the same amount of P4,128.60; that the land has an annual produce worth P400.00; and praying (2) that judgment be rendered: (a) allowing him to repurchase the property for the sum of P4,128,60 and (b) awarding to him P400.00 annually from date of filing of the complaint until the property is delivered to him, with costs. On May 26, 1960, herein petitioners, defendants below, filed their respective answers admitting some material factual allegations in the complaint; but denied the right of private respondent to repurchase the property, and interposed the following affirmative defenses: (1) that at the time the absolute sales were entered into, they were totally ignorant of and had no knowledge whatsoever to any encumbrance or right to repurchase by private respondent, who assured petitioner Francisco Santana that he (Santana) could sell the land in question absolutely and free from any encumbrance and is not subject to any right of repurchase as he (respondent Mariñas) had been in possession of the property for over twenty-five (25) years; (2) that they (petitioners) have always been of the honest belief that they acquired absolute ownership of the property, free from any lien or encumbrances whatsoever and, hence, are purchasers in good faith; (3) that being innocent purchasers for value, they acquired absolute ownership over the property and private respondent cannot enforce against them any right of repurchase of whatever nature; (4) that as absolute owners and possessors in good faith, they (petitioners) incurred necessary and useful expenses thereon in the total amount of not less than P10,000.00; and (5) that the property in question now a residential area with real estate subdivisions and roads in front and at the back thereof and its present increased value is no less than P2.50 a square meter. Petitioners interposed a counterclaim for moral damages in the amount of P10,000.00 and attorney's fees and litigation expenses in the total sum of P5,000.00. In their prayer petitioners asked for the rendition of judgment absolving them completely from the complaint, with costs, and sentencing private respondent to pay them moral damages of P10,000.00 and attorney's fees and litigation expenses in the amount of P5,000.00; or in the remote possibility that repurchase by private respondent were allowed, to require the latter to pay the reasonable market value of not less than P2.50 per square meter. As stated at the outset, the trial court ruled out private respondent's right to repurchase the property and dismissed the complaint but that on appeal, the Court of Appeals — Special Sixth Division — reversed the trial court's decision of dismissal and ordered petitioners to reconvey the land to private respondent upon payment to the former of "the repurchase price thereof in the amount of P4,128.60, without special pronouncement as to costs." xxx

xxx

xxx

Page | 36

2.

Petitioners next assail the order of the appellate court directing them to reconvey the subject land to private respondent. Put thus in issue is the proper construction and application of Section 119 of the Public Land Law, Com. Act No. 141, which provides: "Every conveyance of land acquired under the free patent or homestead provisions, when proper, shall be subject to repurchase by the applicant, his widow, or legal heirs, within a period of five years from the date of conveyance." Petitioners' specific contention that it could not have been the intention of the Legislature to subject to the right of repurchase a free patent or homestead conveyed 25 years after the issuance of the title is without legal basis and is contrary to jurisprudence laid down on the matter. Thus, in Isaac, et al. v. Tan Chuan Leong, et al. the sale took place more than 27 years after the issuance of the original title; while in Francisco v. Certeza, Sr., one of the 2 lots was sold more than 41 years after it was acquired. The right to repurchase was upheld in both cases despite the fact that the above lots were acquired under Act No. 296 which contained no provision on the right of redemption. For the right of repurchase was provided for only later, under Section 117 of Act No. 2874, approved on Nov. 29, 1919, and incorporated in Com. Act No. 141 as Section 119. However, We uphold petitioners' proposition that to allow the repurchase of the subject land, under the peculiar circumstances obtaining herein, would be repugnant to the philosophy behind Section 119 of C.A. No. 141 and the  jurisprudence laid down on the matter. The findings of fact of the trial court — the then CFI Judge, Cecilia Muñoz Palma, later a member of this Court, presiding — are clear and duly supported by the evidence. We quote: "Evidence has been adduced by the defendants that this property of Sotero Mariñas has ceased to be in the nature of a homestead, and that instead it has been transformed into a growing commercial and residential area. The vicinity of the property is now a vast expanding business empire, the lands having (been) converted into subdivisions which are sold to the public at fantastic prices. Close to this particular property of Sotero Mariñas the subdivision being developed by a son of the plaintiff who has extensive business interests centered on construction of buildings such as the Rizal Provincial Capitol and development of subdivisions. (See Exhs. "1" to "1G"). By plaintiff's own admission on cross-examination he is 78 years old and sick with a lung ailment; while from the testimony of his son, Antonio Mariñas, it is shown that the sons of plaintiff are all financially independent from the latter and have their respective properties and means of livelihood. Under these circumstances it is evident that to grant plaintiff the right to repurchase the property at this time would be not for the purpose of giving him back the land for his house and cultivation but for him to exploit it for business purposes at the expense of the defendants who are innocent purchaser(s) in good faith and for value." In Simeon vs. Peña We analyzed the various cases previously decided, and arrived at the conclusion that the plain intent, the raison d tre, of Section 119, C.A. No. 141 " . . .is to give the homesteader or patentee every chance to preserve for himself and his family the land that the state had gratuitously given to him as a reward for his labor in cleaning and cultivating it." In the same breath, We agreed with the trial court, in that case, that "it is in this sense that the provision of law in question becomes unqualified and unconditional. And in keeping with such reasons behind Page | 37

the passage of the law, its basic objective is to promote public policy, that is, to provide home and decent living for destitutes, aimed at promoting a class of independent small landholders which is the bulwark of peace and order." As it was in Simeon v. Peña, respondent Mariñas' intention in exercising the right of repurchase "is not for the purpose of preserving the same within the family fold", but "to dispose of it again for greater profit in violation of the law's policy and spirit." The foregoing conclusions are supported by the trial court's findings of fact already cited, culled from evidence adduced. Thus respondent Marinas was 71 years old and a widower at the time of the sale in 1956; that he was 78 when he testified on Oct. 24, 1963 (or over 94 years old today if still alive); that . . . he was not living on the property when he sold the same but was residing in the poblacion attending to a hardware store; and that the property was no longer agricultural at the time of the sale, but was a residential and commercial lot in the midst of many subdivisions. The profit motivation behind the effort to repurchase was conclusively shown when the then plaintiff's counsel, in the case below, Atty. Loreto Castillo, in his presence, suggested to herein petitioners' counsel, Atty. Rafael Dinglasan, " . . . to just add to the original price so the case would be settled." Moreover, Atty. Castillo manifested in court that an amicable settlement was possible, for which reason he asked for time "within which to settle the terms thereof" and that "the plaintiff . . . Mr. Mariñas, has manifested to the Court that if the defendants would be willing to pay the sum of One Peso and Fifty Centavos (P1.50) per square meter, he would be willing to accept the offer and dismiss the case." Respondent Mariñas' admission is on record that the money with which he would repurchase the property was not his but belonged to his children one of whom is Felix Mariñas, owner of Cristimar Subdivision. Furthermore, the trial court found that Mariñas' ones "are all financially independent from the latter and have their respective properties and means of livelihood." The respondent Court of Appeals anchors respondent-appellee Mariñas' right of repurchase on "old age and tuberculosis having caught up with appellant, and the land in question being his only property." Allowing the repurchase would, thus, "help tide over the needs of his remaining days." according to respondent court. It could be true that the land in question is the only land owned by respondentappellee. But this is not the determinant factor in allowing the repurchase of land acquired through homestead or free patent. The doctrine in Simeon v. Peña, supra, is explicit that what is "unqualified and unconditional" is the right of the homesteader or patentee to preserve the land "for himself and his family. We can, therefore, properly inquire into the motives behind the repurchase and convinced as We are in the instant case, that the intention is not so, but to exploit it for business purposes or greater profit, We can deny the repurchase. To sustain respondent-appellee's claim under the circumstances would put a premium on speculation contrary to the philosophy behind Sec. 119 of Com. Act No. 141, otherwise known as the Public Land Law. Thus, this Court, speaking through Mr.  Justice J.B.L. Reyes, held in Santander, et al. v. Villanueva that the law discourages homesteaders from taking advantage of the "salutary policy behind the Public Land Law to enable them to recover the land in question from (vendees) only to dispose of it again at much greater profit to themselves." xxx

xxx

xxx Page | 38

View more...

Comments

Copyright ©2017 KUPDF Inc.
SUPPORT KUPDF